Sunteți pe pagina 1din 48

Cardiology I

Cardiology I
Shannon W. Finks, Pharm.D., FCCP,
BCPS (AQ Cardiology)
Associate Professor
University of Tennessee College of Pharmacy
Clinical Pharmacy Specialist-Cardiology
Veterans Affairs Medical Center Memphis
Memphis, Tennessee

ACCP Updates in Therapeutics 2013: The Pharmacotherapy Preparatory Review and Recertification Course
2-205

Cardiology I

Learning Objectives
1. Distinguish between the acute coronary syndromes
ST-segment elevation myocardial infarction, non
ST-segment elevation myocardial infarction, and
unstable anginaby diagnosis and treatment.
2. Formulate evidence-based treatment strategies for
patients with acute decompensated heart failure.
3. Differentiate between goals and treatment for hypertensive emergencies and hypertension without
progressive organ damage.
4. Devise a treatment plan for patients presenting
with a life-threatening arrhythmia.
5. Provide evidence-based treatment for a patient
given a diagnosis of idiopathic pulmonary arterial
hypertension.

2. A 56-year-old man presents to the hospital with


the chief concern of chest pain that was unrelieved
at home with nitroglycerin. His ECG shows STsegment depression and T-wave inversion. Cardiac
markers are drawn that show an elevated troponin
I. The cardiologist has requested that the patient go
to the cardiac catheterization laboratory for further
evaluation. The patient has a history of coronary
artery disease (CAD) and had a myocardial infarction (MI) about 6 months ago. During his previous
hospitalization, the patient was thought to have developed heparin-induced thrombocytopenia (HIT)
because his platelet count dropped to 40 x 103 cells/
mm3 after his previous catheterization. Given this
patients diagnosis and history, which treatment
regimen would be most appropriate to use during
his cardiac catheterization?
A. Abciximab.
B. Bivalirudin.
C. Eptifibatide.
D. Tenecteplase.

Self-Assessment Questions
Answers and explanations to these questions
can be found at the end of this chapter.
1. A 62-year-old man presents to the emergency department with the chief concern of chest pain that
woke him from sleep and radiates to his jaw. An
electrocardiogram (ECG) is conducted, which reveals ST-segment depression in leads V2V4. His
blood pressure (BP) is 112/62 mm Hg, and heart
rate (HR) is 90 beats/minute. Cardiac enzymes
have been drawn, and the first troponin result returned slightly positive. Preparations are under
way to take the patient to the cardiac catheterization laboratory for evaluation. Which medication
regimen represents the most appropriate treatment
for this patient at this time?
A. Aspirin (ASA) 325 mg, clopidogrel 600 mg
loading dose, and unfractionated heparin
(UFH) infusion 80-unit/kg bolus, followed
by 18 units/kg/hour and metoprolol 5 mg
intravenously x 3.
B. ASA 81 mg; prasugrel 60-mg loading dose;
UFH infusion 60-unit/kg bolus, followed by
12 units/kg/hour; and intravenous enalaprilat.
C. ASA 325 mg, ticagrelor 180-mg loading dose,
and UFH infusion 60-unit/kg bolus, followed
by 12 units/kg/hour.
D. ASA 81 mg, clopidogrel 600 mg,
nitroglycerin infusion at a rate of 10 mcg/
minute, and bivalirudin 0.75-mg/kg bolus and
1.75-mg/kg/hour infusion.

3. A 60-year-old woman with New York Heart Association (NYHA) class IV heart failure (HF) is
admitted for increased shortness of breath and dyspnea at rest. Her extremities appear well perfused,
but she has 3+ pitting edema in her lower extremities. Her vital signs include BP 125/70 mm Hg, HR
92 beats/minute, and O2 saturation 89% on 100%
facemask. After initiating an intravenous diuretic,
which is the best intravenous drug to treat this
patient?
A. Dobutamine.
B. Milrinone.
C. Nitroglycerin.
D. Metoprolol.
4. A 75-year-old woman has a history of NYHA class
III HF (left ventricular ejection fraction [LVEF]
25%) and several nonST-segment elevation myocardial infarctions (NSTEMIs). She had an episode
of sustained ventricular tachycardia (VT) during
this hospitalization for pneumonia. Her corrected
QT (QTc) interval was 380 milliseconds on the
telemetry monitor, and her serum potassium and
magnesium were 4.6 mEq/L and 2.2 mg/dL, respectively. Which intravenous agent is most appropriate for this patient?

ACCP Updates in Therapeutics 2013: The Pharmacotherapy Preparatory Review and Recertification Course
2-206

Cardiology I

A. Procainamide.
B. Metoprolol.
C. Magnesium.
D. Amiodarone.

A. Propafenone.
B. Amiodarone.
C. Implantable cardioverter defibrillator (ICD).
D. Metoprolol.

5. A 53-year-old woman is admitted to the hospital


after the worst headache she has ever experienced.
Her medical history includes exertional asthma,
poorly controlled hypertension (HTN), and hyperlipidemia. She is nonadherent to her medications,
and she has not taken her prescribed BP medications for 4 days. Vital signs include BP 220/100
mm Hg and HR 65 beats/minute. She is found to
have a cerebrovascular accident. Which agent is
most appropriate for this patients hypertensive
emergency?
A.
B.
C.
D.

Fenoldopam 0.1 mcg/kg/minute.


Nicardipine 5 mg/hour.
Labetalol 0.5 mg/minute.
Enalaprilat 0.625 mg intravenously every 6
hours.

6. A 56-year-old African American man has a long


history of poorly controlled HTN secondary to
medication nonadherence and subsequent dilated
cardiomyopathy (LVEF 35%). He is assessed in a
community health clinic today and reports not having taken his medications for the past week. The
patient is asymptomatic, and his examination is
unremarkable except for BP 180/120 mm Hg and
HR 92 beats/minute. All laboratory values are
within normal limits except for a serum creatinine
(SCr) of 1.4 mg/dL and urinalysis with 2+ proteinuria. Which regimen would be best to manage this
patient in the clinic?
A.
B.
C.
D.

Nifedipine 10 mg sublingually.
Clonidine 0.2 mg orally.
Captopril 12.5 mg orally.
Labetalol 200 mg orally.

7. A 52-year-old woman experiences a witnessed cardiac arrest in a shopping mall; she is resuscitated
with an automatic external defibrillator device.
On electrophysiologic study, she has inducible
VT. Which agent is most appropriate for reducing
the secondary incidence of sudden cardiac death
(SCD)?

8. The Sudden Cardiac Death in Heart Failure trial


evaluated the efficacy of amiodarone or an ICD
versus placebo in preventing all-cause mortality in
ischemic and nonischemic NYHA class II and III
patients with HF. There was a 7.2% absolute risk
reduction and a 23% relative risk reduction in allcause mortality at 60 months with an ICD versus
placebo. Which best shows the number of patients
needed to treat with an ICD to prevent one death
versus placebo?
A. 1.
B. 4.
C. 14.
D. 43.
9. You are working on a review article about newer treatment strategies for hypertensive crises.
You want to ensure that you retrieve all relevant
clinical trials and related articles on your subject.
Which comprehensive database is most appropriate to search to ensure that you have not missed key
articles?
A. International Pharmaceutical Abstracts.
B. Iowa Drug Information Service.
C. Clin-Alert.
D. Excerpta Medica.
10. A physician on your team asks that you report an
adverse drug reaction (ADR) experienced by a patient taking nesiritide. The patient had severe hypotension after the initial bolus dose of nesiritide,
even though his BP was in the normal range before
therapy initiation. The hypotension led to reduced
renal perfusion, resulting in oliguric acute kidney
injury and subsequent hemodialysis. The patient
had no known renal insufficiency before developing this complication. Which statement best
describes the Joint Commission requirements for
institutional ADR reporting?
A. A MedWatch form must be completed that
explains the situation in which the ADR
occurred.

ACCP Updates in Therapeutics 2013: The Pharmacotherapy Preparatory Review and Recertification Course
2-207

Cardiology I

B. Institutions must create their own definition


of ADR with which practitioners will be
familiar.
C. Hospital staff must use the Naranjo algorithm
for assessing the severity of the ADR.
D. Only severe or life-threatening ADRs need to
be reported.
11. Your Pharmacy and Therapeutics Committee
wants you to perform a pharmacoeconomic analysis of a new drug available to treat decompensated
HF. This drug works through a unique mechanism
of action. Unlike other available inotropic therapies that can increase mortality, this drug appears
to reduce long-term mortality. However, the cost is
10 times greater than that of other available drugs.
Which pharmacoeconomic analysis would be best
to determine whether this new drug is a better formulary choice than currently available agents?
A.
B.
C.
D.

Cost-minimization analysis.
Cost-effectiveness analysis.
Cost-benefit analysis.
Cost-utility analysis.

ACCP Updates in Therapeutics 2013: The Pharmacotherapy Preparatory Review and Recertification Course
2-208

Cardiology I

I. ACUTE CORONARY SYNDROMES


A. Definitions and Goals
1. Definitions
Table 1. ACS Definition
Subjective Findings
Unstable angina
(UA)

NonST-segment
elevation MI
(NSTEMI)

ST-segment
elevation MI
(STEMI)

Objective Findings

Pain can be similar to typical


angina, but can be more severe
and can occur at rest, with less
exertion than typical angina.
Chest pain can be new in onset,
or it can present like existing
angina with greater intensity,
duration, and frequency. May
worsen suddenly or may be
stuttering in nature, recurring
over days to weeks

ST-segment depression,
T-wave inversion, or
transient or nonspecific
ECG changes may occur

Atypical symptomsa (without


chest pain) can occur that
include arm pain, lower jaw
pain, back pain, shortness of
breath, diaphoresis, or angina
equivalents such as dyspnea or
extreme fatigue

Positive biomarkers
(troponin I or T elevation,
CK-MB fraction > 5% of
total CK)
ST-segment elevation of
> 1 mm above baseline
on ECG in two or more
contiguous leads

Classic symptoms include


worsening of pain or pressure in
chest, characterized as viselike,
suffocating, squeezing, aching,
gripping, and excruciating,
that may be accompanied by
radiation

No positive biomarkers for


cardiac necrosis present
ST-segment depression,
T-wave inversion, or
transient or nonspecific
ECG changes may occur

Extent of Injury
No myocardial
injury; partial
occlusion of coronary
artery

Myocardial injury;
partial occlusion of
coronary artery

Myocardial necrosis;
total occlusion of
coronary artery

Positive biomarkers
(troponin I or T elevation,
CK-MB fraction > 5% of
total CK)

Note: Up to one-half of all MIs are silent or unrecognized, and one-third present with symptoms other than chest discomfort.
ACS = acute coronary syndromes; CK = creatine kinase; CK-MB = creatine kinase myocardial band; ECG = electrocardiogram; MI = myocardial
infarction.
a

2. Therapy goals
a. Unstable angina (UA)/NSTEMI goals
i. Prevent total occlusion of the infarct-related artery.
(a) Medical management includes antiplatelet therapy (ASA plus P2Y12 receptor inhibitor
plus or minus glycoprotein IIb/IIIa inhibitors [GPIs]) and antithrombotic therapy (UFH,
low-molecular-weight heparin [LMWH], fondaparinux, or bivalirudin)
(b) Percutaneous coronary intervention (PCI):
(1) Percutaneous transluminal coronary angioplasty (i.e., balloon)
(2) Stent implantation (bare metal [BMS] or drug-eluting [DES])
(3) Arthrectomy

ACCP Updates in Therapeutics 2013: The Pharmacotherapy Preparatory Review and Recertification Course
2-209

Cardiology I

(c) Thrombolytics have no role and have an increased bleeding risk.


ii. Control chest pain and associated symptoms.
b. ST-segment elevation myocardial infarction (STEMI) goals
i. Restore patency of the infarct-related artery and minimize infarct size.
(a) Thrombolytic medications with door-to-needle time within 30 minutes
(b) Primary PCI with door-to-balloon time within 90 minutes
(1) If presenting to a facility without the capability for expert, prompt intervention with
primary PCI within 90 minutes, should undergo fibrinolysis unless contraindicated
(2) Facilitated PCI consists of planned, immediate PCI after an initial pharmacologic
regimen such as full- or half-dose fibrinolysis, a GPI, or a combination of such;
might be performed as a reperfusion strategy in higher-risk patients when PCI is not
immediately available and bleeding risk is low
(3) Rescue PCI consists of PCI after failed thrombolysis and is indicated in select
patients if shock, severe HF, and/or pulmonary edema, hemodynamic or electrical
instability, evidence of persistent ischemia
ii. Prevent complications such as arrhythmias or death.
iii. Control chest pain and associated symptoms. Nonsteroidal anti-inflammatory drugs (NSAIDs)
(both nonselective and cyclooxygenase 2 [COX-2]) should be discontinued (increased risk of
mortality, reinfarction, HTN, HF, and myocardial rupture).
B. Invasive or Conservative Treatment Strategy
1. Calculate thrombolysis in myocardial infarction (TIMI) score (see Table 2) by adding 1 point for each
history or presentation finding.
2. Risk of mortality, new or recurrent MI, or severe recurrent ischemia through 14 days on the basis of
TIMI score: score = 01, mortality 7%; score = 2, mortality 8%; score = 3, mortality 13%; score = 4,
mortality 20%; score = 5, mortality 26%; and score = 67, mortality 41%

Table 2. TIMI Risk Score for Unstable Angina/NSTEMIa,b


Historical
Age > 65
Three cardiac risk factors (hypertension, diabetes,
hyperlipidemia, smoking, family history)
Known coronary artery disease 50% stenosis

Points
1
1
1

Presentation
Severe angina ( 2 episodes within 24 hours
ASA within 7 days
Elevated markers
ST-segment deviation > 0.5 mm

1
1
1
1

Risk of mortality, new or recurrent MI, or severe recurrent ischemia through 14 days. Low is 02, intermediate is 3, and > 4 is high risk.
Risk score = total points (07).
ASA = aspirin; MI = myocardial infarction; NSTEMI = nonST-segment elevation myocardial infarction; TIMI = thrombolysis in myocardial
infarction.
a

ACCP Updates in Therapeutics 2013: The Pharmacotherapy Preparatory Review and Recertification Course
2-210

Cardiology I

Table 3. Selection of Initial Treatment Strategy: Invasive vs. Conservative


Initial Invasive Strategy
Recurrent angina/ischemia at rest with low-level activities despite intensive medical therapy
Elevated cardiac biomarkers (TnT or TnI)
New/presumably new ST-segment depression
Signs/symptoms of HF or new/worsening mitral regurgitation
High-risk findings from noninvasive testing
Hemodynamic instability
Sustained VT
PCI within 6 months
Prior CABG
High-risk score (e.g., TIMI, GRACE)
Reduced left ventricular function (LVEF < 40%)
Initial Conservative Strategy
Low-risk score (e.g., TIMI, GRACE); high risk of catheterization-related complications; not a good candidate for
revascularization; or patient/physician presence in the absence of high-risk features
CABG = coronary artery bypass grafting; GRACE = Global Registry of Acute Coronary Events; HF = heart failure; LVEF = left ventricular
ejection fraction; PCI = percutaneous coronary intervention; TIMI = thrombolysis in myocardial infarction; TnI = troponin I; TnT = troponin
T; VT = ventricular tachycardia.

C. Initial Management
1. MONA plus -blocker
Table 4. Initial Management of ACS MONA plus -Blocker
M = Morphine Morphine 24 mg IV q515 minutes is reasonable if symptoms are not relieved despite NTG or
if symptoms recur
O = Oxygen

Oxygen if O2 saturation < 90% or high-risk features of hypoxemia

N=
Nitroglycerin

NTG spray or SL tablet 0.4 mg 3 to relieve acute chest pain (if pain is unrelieved after one dose,
call 911)
NTG IV 510 mcg/minute, titrate to chest pain relief or 200 mcg/minute if pain unrelieved by
morphine and SL NTG
- Used in first 48 hours for treatment of persistent chest pain, HF, and HTN
- Use should not preclude other mortality-reducing therapies (-blocker, ACE inhibitor)
- No mortality benefits but high placebo crossover rate
CIs: Sildenafil or vardenafil use within 24 hours or tadalafil use within 48 hours; SBP < 90 mm
Hg or 30 mm Hg below baseline, HR < 50 beats/minute, HR > 100 beats/minute in absence of
symptomatic HF or right ventricular infarction

A = Aspirin

ASA chew and swallow nonenteric coated 162325 mg 1


- Clopidogrel If ASA allergy or considerable gastrointestinal intolerance
- Performance measure

ACCP Updates in Therapeutics 2013: The Pharmacotherapy Preparatory Review and Recertification Course
2-211

Cardiology I

Table 4. Initial Management of ACS MONA plus -Blocker (continued)


-Blocker

Oral or IV -blocker (oral route preferred, class I)


- Mortality benefit in early phases of acute MI but also if initiated later
- Performance measure within 24 hours
- IV route reasonable if tachyarrhythmia or HTN present (class IIa)
- Avoid agents with ISA
CIs: Hypotension, signs of HF, risk factors for cardiogenic shock, or other relative
contraindications (third-degree heart block, active asthma)

Other early hospital therapies


ACE
inhibitors

Indicated orally within first 24 hours if anterior MI, HF, LVEF 40%, unless CI
- IV therapy contraindicated because of risk of hypotension
Consider in all patients post-ACS unless CI
Continued indefinitely in all patients post-ACS with LVEF < 40%, HF, hypertension, or diabetes
ARB indicated if contraindication to ACE inhibitor
CIs: Hypotension, pregnancy, bilateral renal artery stenosis

Aldosterone
antagonist

Indicated in patients post-ACS already receiving ACE inhibitor and -blocker and who have
LVEF 40% and either symptomatic HF or diabetes, unless CIs
CIs: Hyperkalemia (K > 5.0), CrCl < 30 mL/minute

Statins

High-intensity statin therapy should be initiated or continued in all patients without CIs

CCBs

Specifically, nondihydropyridine CCBs Verapamil, diltiazem


Recommended if continuing or frequently recurring ischemia and contraindication to -blocker
therapy or recurrent ischemia after -blockers and nitrates fully used
- No real benefit or detriment to mortality; primarily symptom relief effects
CIs: Clinically significant LV dysfunction; immediate-release dihydropyridine CCBs should not
be administered in the absence of a -blocker

ACE = angiotensin-converting enzyme; ACS = acute coronary syndrome; ARB = angiotensin receptor blocker; ASA = aspirin; CAD = coronary
artery disease; CCB = calcium channel blocker; CKD = chronic kidney disease; CI= contraindication; HF = heart failure; HTN = hypertension;
ISA = intrinsic sympathomimetic activity; IV = intravenous(ly); K = potassium; LV = left ventricular; LVEF = left ventricular ejection fraction;
MAP = mean arterial pressure; MI = myocardial infarction; N/A = not available; NTG = nitroglycerin; q = every; SBP = systolic blood pressure;
SL = sublingual.

ACCP Updates in Therapeutics 2013: The Pharmacotherapy Preparatory Review and Recertification Course
2-212

Cardiology I

2. Antiplatelet and anticoagulant strategies for acute coronary syndromes (ACS) (see Tables 5 and 6)
Table 5. Antiplatelet and Anticoagulant Management Strategies in UA/NSTEMI
Strategy

Invasive Strategy

Initial Conservative Strategy or Unknown

Anticoagulant
therapya

Class I agents: Enoxaparin (LOE: A), UFH


(LOE: A), fondaparinuxb (LOE: B), or
bivalirudin (LOE: B)

Class I agents: Enoxaparin (LOE: A), UFH


(LOE: A), or fondaparinux (LOE: B)

Aspirin PLUS:

Aspirin PLUS:

Class I agents: Clopidogrel (LOE: A) or


prasugreld (LOE: B) or ticagrelord (LOE: B)
(+ selective GPIe)

Class I agents: Clopidogrel (LOE: B) or


ticagrelor (LOE: B)

Antiplatelet
therapy

(Enoxaparin or fondaparinux may be


preferred over UFH)c

If high risk of bleeding, fondaparinux or bivalirudin preferred. If CABG planned, UFH preferred.
Fondaparinux should not be used as the sole anticoagulant during PCI. It is given a class I recommendation in the 2012 UA/NSTEMI guidelines
and a class III or harmful recommendation in the 2011 PCI guidelines.
c
Enoxaparin or fondaparinux in preference to UFH is given a class IIa LOE B recommendation in the 2012 UA/NSTEMI guidelines.
d
If unlikely to undergo CABG, initiate prasugrel or ticagrelor at time of PCI.
e
Benefit from adding GPI to dual antiplatelet therapy (DAT) is greatest among those with highest-risk features (elevated biomarkers, diabetes,
those undergoing revascularization). GPIs were studied in those receiving UFH as anticoagulant strategy. After PCI, discontinue anticoagulation
and continue GPI for 1224 hours at the discretion of the physician.
CABG = coronary artery bypass grafting; GPI = glycoprotein IIb/IIIa inhibitor; LOE = level of evidence; NSTEMI = nonST-segment elevation
myocardial infarction; NTG = nitroglycerin; PCI = percutaneous coronary intervention; UA = unstable angina; UFH = unfractionated heparin.
a

Table 6. Antiplatelet and Anticoagulant Management Strategies in STEMI


Treatment Option
Anticoagulant therapy

Primary PCI
Class I agents: IV UFH or
bivalirudin

Antiplatelet therapy

Aspirin PLUS:

Fibrinolysis
Should receive anticoagulant therapy with
fibrinolysis for a minimum of 48 hours,
preferably for the duration of hospitalization or
until revascularization. Regimens include UFH,
enoxaparin, and fondaparinuxa
Aspirin PLUS:

Clopidogrel, prasugrel, or ticagrelor


(All class I: LOE: B) selective
GPI use

Clopidogrel preferred selective GPI use

Fondaparinux should not be used as the sole anticoagulant to support PCI. Give additional anticoagulant during revascularization if fondaparinux
was initially chosen as anticoagulant strategy.
GPI = glycoprotein IIb/IIIa inhibitor; IV = intravenous; LOE = level of evidence; PCI = percutaneous coronary intervention; SC = subcutaneous;
STEMI = ST-segment elevation myocardial infarction; UFH = unfractionated heparin.
a

ACCP Updates in Therapeutics 2013: The Pharmacotherapy Preparatory Review and Recertification Course
2-213

Cardiology I

3. Antiplatelet and anticoagulant therapy (see Tables 710)


Table 7. Comparison of P2Y12 Receptor Inhibitors
Parameter
FDA indication
Peak platelet inhibition
Dosing

Metabolism

Half-life
Nonresponders

Drug-disease
interactions

CABG hold time


Risk of bleeding

Clopidogrel (Plavix)a
ACS managed medically
or with PCI
300-mg load 6 hours
600-mg load 2 hours
LD: 300600 mg
MD: 75 mg daily
Prodrug; converted
by two-step process
involving 2C19 and
3A4
8 hours metabolite
Exposure to active drug
affected by CYP2C19
and CYP3A4 and PGP
polymorphisms
PPIs inhibit 2C19
(concomitant use
with omeprazole
is discouraged per
package labeling;
enhanced bleeding with
NSAIDs, VKA, O3FA,
etc.
5 days
Less than PRA and TIC
with standard dosing

Box warning

Genetic polymorphisms

Study

CREDO, CURE, PCICURE, CLARITY,


COMMIT

Prasugrel (Effient)b
ACS with PCI
60-mg load ~11.5 hoursd

Ticagrelor (Brilinta)c
ACS managed medically or with
PCI
180-mg load < 1 hourd

LD: 60 mg
MD: 10 mg daily (5 mge if <
60 kg; BW 75 years)
CIs: TIA/stroke
Prodrug; converted to active
metabolite by several CYP
pathways

LD: 180 mg
MD: 90 mg twice daily
CIs: ICH, severe hepatic disease

3.7 hours metabolite (range


215 hours)
No known issues

7 hours (parent), 9 hours (active


metabolite)
No known issues

Not prodrug; reversible,


noncompetitive binding; 3A4
(primary), 3A5, P-gp

Less prone, but data are


Careful with asthma, bradycardia:
limited; enhanced bleeding Enhanced bleeding with NSAIDs,
with NSAIDs, VKA, etc.
VKA; strong 3A4 inhibitors
ticagrelor concentrations;
strong 3A4 inducers ticagrelor
concentrations; do not exceed 40
mg of simvastatin or lovastatin;
limit ASA to < 100 mg
7 days
5 days
Spontaneous, instrumented, Risk of non-CABG bleeds higher
and fatal bleeds higher than than standard-dose CLO
standard-dose CLO
Age-related bleeding
Aspirin dosing > 100 mg
CVA
TRITON-TIMI 38
PLATO

Discontinue CLO at least 5 days before surgery. Administer CLO indefinitely if ASA allergy. Avoid LD if patient is 75 years or older in STEMI
when fibrinolysis is given.
b
Discontinue PRA at least 7 days before surgery. Avoid PRA in patients with active pathologic bleeding or a history of TIA or stroke as well as in
patients older than 75 years unless patient has DM or history of myocardial infarction.
c
Discontinue TIC at least 5 days before surgery. Avoid TIC in patients with active pathologic bleeding or a history of ICH. Avoid doses of ASA >
100 mg daily (exception: first dose of 325 mg).
d
A significant antiplatelet effect has been observed at 30 minutes. Both onset of effect and extent of platelet inhibition is greater than seen with CLO.
e
Although using 5 mg in patients weighing < 60 kg should be considered, this dose has not been prospectively studied.
ACS = acute coronary syndromes; ASA = aspirin; BW = box warning; CABG = coronary artery bypass grafting; CI = contraindication; CLO =
clopidogrel; CVA = cerebrovascular accident; CYP = cytochrome P450; DM = diabetes mellitus; ICH = intracranial hemorrhage; LD = loading dose;
MD = maintenance dose; NSAID = nonsteroidal anti-inflammatory drug; O3FA = omega-3 fatty acids; PCI = percutaneous coronary intervention;
P-gp = P-glycoprotein; PPI = proton pump inhibitor; PRA = prasugrel; STEMI = ST-segment elevation myocardial infarction; TIA = transient
ischemic attack; TIC = ticagrelor; VKA = vitamin K antagonist.
a

ACCP Updates in Therapeutics 2013: The Pharmacotherapy Preparatory Review and Recertification Course
2-214

Cardiology I

Table 8. Dual Antiplatelet Therapy Dosing Strategies in ACS with or without PCI
ASA

P2Y12 Receptor Inhibitor


Initial ACS therapy

ASA 162325 mg nonenteric


orally or chewed 1

STEMI
CLO 600-mg LD or PRA 60-mg LD or TIC 180-mg LD for primary PCI
Pre-PCI after fibrinolytic therapy 300 CLO 300-mg LD if within 24 hours
of event; CLO 600-mg LD if > 24 hours after event
UA/NSTEMI
CLO 600-mg LD or PRA 60-mg LD or TIC 180-mg LD if PCI planned
CLO 600-mg LD or TIC 180-mg LD if medical management
Pre-PCI

Already on ASA therapy,


81325 mg
Not on ASA therapy, ASA 325
mg before PCI

CLO 600-mg LD as soon as possible before or at time of PCI or


PRA 60-mg LD promptly or no later than 1 hour after coronary
anatomy defined or
TIC 180-mg LD as early as possible before or at time of PCI
Post-PCI (ACS indication)

Continue ASA indefinitely 81


mg daily

CLO 75 mg daily or PRA 10 mg dailya or TIC 90 mg twice daily for a


minimum of 12 months
Post-elective PCI (no ACS)

BMS or DES:
ASA 81 mg/day indefinitely

BMS:
CLO 75 mg/day for a minimum of 1 month and ideally up to 12 monthsb
DES:
CLO 75 mg/day for a minimum of 12 months
Post-ACS (medical management; no PCI)

ASA 75162 mg/day continued


indefinitely

CLO 75 mg/day or TIC 90 mg twice daily for up to 12 monthsb

If patient < 60 kg, use 5 mg daily per package labeling.


Earlier discontinuation can be considered if the risk of bleeding outweighs the benefit.
ACS = acute coronary syndromes; ASA = aspirin; BMS = bare metal stent; CABG = coronary artery bypass grafting; CLO = clopidogrel; DES =
drug-eluting stent; DM = diabetes mellitus; LD = loading dose; NSTEMI = nonST-segment elevation myocardial infarction; PCI = percutaneous
coronary intervention; PRA = prasugrel; STEMI = ST-segment elevation myocardial infarction; TIC = ticagrelor.
a

ACCP Updates in Therapeutics 2013: The Pharmacotherapy Preparatory Review and Recertification Course
2-215

Cardiology I

Table 9. GP IIb/IIIa Inhibitor Dosing in ACS with or without PCIa


Pretreated with P2Y12
Abciximab
(ReoPro)b

Of uncertain benefit

Eptifibatide Of uncertain benefit


(Integrilin)

Not Pretreated with P2Y12


PCI: 0.25-mg/kg IV bolus; then 0.125
mcg/kg/minute (maximum 10 mcg/kg)
for 12 hours

Renal Adjustments
Renal adjustment not
necessary

ACS without PCI: Not recommended


PCI: 180-mcg/kg IV bolus 2 (10
If CrCl < 50 mL/minute,
minutes apart); 2 mcg/kg/minute
reduce infusion 50%;
started after first bolus for 1824 hoursc avoid in patients on
hemodialysis; not studied
ACS without PCI: Of uncertain benefit
in patients with SCr > 4
in patients adequately pretreated with
mg/dL
a P2Y receptor inhibitor; single bolus
12

Tirofiban
(Aggrastat)

Of uncertain benefit

used as above
PCI: 25-mcg/kg IV bolus over 3
minutes; then 0.15 mcg/kg/minute for
1824 hours

If CrCl < 30 mL/minute,


reduce infusion 50%

ACS without PCId: 0.4 mcg/kg/minute


for 30 minutes (LD infusion); then 0.1
mcg/kg/minute for 1872 hours
All agents should be used in combination with heparin (either UFH or LMWH) and ASA.
Not recommended in those not undergoing PCI because of clinical trial results.
c
Double bolus is recommended to support PCI in STEMI and UA/NSTEMI.
d
Lower dose used to treat medically managed UA/NSTEMI or when there is substantial delay to PCI (i.e., 48 hours).
ACS = acute coronary syndromes; CrCl = creatinine clearance; GP = glycoprotein; IV = intravenous; LD = loading dose; LMWH = lowmolecular-weight heparin; NSTEMI = nonST-segment elevation myocardial infarction; PCI = percutaneous coronary intervention; SCr =
serum creatinine; STEMI = ST-segment elevation myocardial infarction; UA = unstable angina; UFH = unfractionated heparin.
a

Table 10. Antithrombotic Dosing in ACS with or without PCI


Unfractionated
Heparin

Enoxaparin
(Lovenox)

Fondaparinux
(Arixtra)

Classification

LMWH

Factor Xa inhibitor

UA/NSTEMI

60-unit/kg IVB
(maximum 4000 units),
12 units/kg/hour IV
(maximum 1000 units/
hour) for 48 hours or
end of PCI, goal aPTT
of 5070 seconds
Supplemental doses to
target ACTa

PCI

If GPI, UFH 50- to 70unit/kg IVB


If no GPI, UFH 70- to
100-unit/kg IVB

Bivalirudin
(Angiomax)
DTI

1 mg/kg SC bid
for 2448 hours
or until end of
PCI or throughout
hospitalization (up to
8 days)

2.5 mg SC daily

0.1-mg/kg IVB; then


0.25 mg/kg/hour IV (not
a class I indication for
conservative strategy)

If last dose < 8 hours,


nothing additional
needed
If last dose > 8 hours,
0.3-mg/kg IVB if
last dose 812 hours
prior or fewer than
two therapeutic doses
received before PCI

Fondaparinux
should not be
used as a sole
anticoagulant for
PCI

0.75-mg/kg IVB, 1.75


mg/kg/hour IV
Discontinue at end of
PCI or continue for up to
4 hours postprocedure if
needed
Hold UFH 30 minutes
before administration

ACCP Updates in Therapeutics 2013: The Pharmacotherapy Preparatory Review and Recertification Course
2-216

Cardiology I

Table 10. Antithrombotic Dosing in ACS with or without PCI (continued)


Unfractionated
Enoxaparin
Fondaparinux
Heparin
(Lovenox)
(Arixtra)
STEMI
Supplemental doses to
30 mg IVB, followed 2.5-mg IVB; then
primary PCI
target ACTa
immediately by 1
2.5 mg SC daily
mg/kg SC bid; not to
If GPI, UFH 50- to 70- exceed 100 mg on first
unit/kg IVB
two doses
If no GPI, UFH 70- to
100-unit/kg IVB
If > 75 years, omit
bolus; 0.75 mg/kg SC
bid; not to exceed 75
mg on first two doses
Dose
Avoid if history of HIT If CrCl < 30 mL/
Avoid if CrCl < 30
adjustments/
minute, 1 mg/kg SC
mL/minute
contraindications
daily
Avoid if history of
HIT

Bivalirudin
(Angiomax)
0.75-mg/kg IVB, 1.75
mg/kg/hour IV

Adjust infusion dose in


severe renal dysfunction
If CrCl < 30 mL/
minute, reduce infusion
to 1 mg/kg/hour. If on
hemodialysis, reduce
infusion to 0.25 mg/kg/
hour

Target ACT 250300 seconds for HemoTec and 300350 seconds for Hemochron without GPI and 200250 seconds in patients given a
concomitant GPI.
ACT = activated clotting time; aPTT = activated partial thromboplastin time; bid = twice daily; CrCl = creatinine clearance; DTI = direct
thrombin inhibitor; GPI = glycoprotein IIb/IIIa inhibitor; HIT = heparin-induced thrombocytopenia; IV = intravenous(ly); IVB = intravenous
bolus; LMWH = low-molecular-weight heparin; NSTEMI = nonST-segment elevation myocardial infarction; PCI = percutaneous coronary
intervention; SC = subcutaneously; STEMI = ST-segment elevation myocardial infarction; UA = unstable angina; UFH = unfractionated heparin.
a

4. Fibrinolytic dosing (see Table 11) and adjunctive therapy


a. In the absence of contraindications, fibrinolytic therapy should be given to patients with STEMI
(class I when onset of ischemic symptoms is within the previous 12 hours) when it is anticipated
that primary PCI can not be performed within 120 minutes of first medical contact with an ideal
door to needle time of less than 30 minutes.
b. Patients undergoing reperfusion with fibrinolytics should receive anticoagulant therapy for a
minimum of 48 hours and preferably for the duration of the index hospitalization, up to 8 days or
until revascularization is performed.
c. Recommended regimens include:
i. UFH administration of a 60-unit/kg bolus (maximum 4000 units) and 12 units/kg/hour
(maximum 1000 units/hour), to obtain an aPTT of 1.52.0 times control (around 5070
seconds)
ii. Enoxaparin 30 mg intravenously (if 75 years or older, omit bolus) followed in 15 minutes by a
1-mg/kg subcutaneous injection (if 75 years or older, 0.75 mg/kg) twice daily for the duration
of index hospitalization, up to 8 days or until revascularization. Maximum 100 mg for the first
two doses. If 75 years or older, maximum 75 mg for the first two doses. If CrCl is less than 30
mL/minute, extend dosing interval to daily administration.
iii. Fondaparinux administered with initial 2.5-mg intravenous dose followed in 24 hours by 2.5mg/day subcutaneous injections (contraindicated if CrCl is less than 30 mL/minute) for the
duration of the index hospitalization, up to 8 days or until revascularization

ACCP Updates in Therapeutics 2013: The Pharmacotherapy Preparatory Review and Recertification Course
2-217

Cardiology I

Table 11. Fibrinolytic Therapy


Alteplase (rt-PA, Activase)

Dosing
15 mg IV; then 0.75 mg/kg over 30 minutes (maximum 50 mg);
then 0.5 mg/kg (maximum 35 mg) over 60 minutes

Reteplase (r-PA, Retavase)

10 units IV; repeat 10 units IV in 30 minutes

Tenecteplase
(TNK-tPA, TNKase)

< 60 kg, 30 mg IV; 6069 kg, 35 mg IV; 7079 kg, 40 mg IV; 8089 kg, 45 mg IV,
> 90 kg, 50 mg IV (about 0.5 mg/kg)

IV = intravenously; r-PA = recombinant plasminogen activator; rt-PA = recombinant tissue plasminogen activator; tPA = tissue plasminogen
activator.

Table 12. Contraindications to Fibrinolytic Therapy


Relative Contraindications
BP > 180/110 mm Hg on presentation or history of
chronic poorly controlled HTN
History of prior ischemic stroke > 3 months prior
Recent major surgery (<3 weeks prior),
Traumatic or prolonged CPR (> 10 minutes)
Recent internal bleeding (within 24 weeks)
Active peptic ulcer
Noncompressible vascular punctures
Pregnancy
Known intracranial pathology (dementia)
Oral anticoagulant therapy

Absolute Contraindications
ANY prior hemorrhagic stroke
Ischemic stroke within 3 months (except in past 4.5 hours)
Intracranial neoplasm or arteriovenous malformation
Active internal bleeding
Aortic dissection
Considerable facial trauma or closed-head trauma in past 3
months
Intracranial or intraspinal surgery within 2 months
Severe uncontrolled hypertension (unresponsive to
emergency therapy)
For streptokinase,a treatment within previous 6 months (if
considering streptokinase again)

Streptokinase is no longer marketed in the United States but is available in other countries.
BP = blood pressure; CPR = cardiopulmonary resuscitation; CVA = cerebrovascular accident; HTN = hypertension; INR = international
normalized ratio; TIA = transient ischemic attack.
a

D. Long-term Management
1. -Blockers
a. Indicated for all patients unless contraindicated
b. Initiate within a few days of event, if not acute, and continue for at least 3 years.
c. If moderate or severe left ventricular (LV) failure, initiate with gradual titration.
2. Angiotensin-converting enzyme (ACE) inhibitors
a. Indicated for all patients even if no LV dysfunction, HTN, or diabetes mellitus (DM)
b. Give oral ACE inhibitor in low doses to all patients during the first 24 hours of anterior STEMI,
HF signs (pulmonary congestion), or LVEF less than 40%, provided no hypotension exists
(systolic BP less than 100 mm Hg) or other contraindication, to reduce mortality and remodeling.
c. Angiotensin receptor blocker if ACE inhibitor intolerant
d. Avoid intravenous ACE inhibitor post-MI to prevent hypotension.
3. Aldosterone receptor blockers: Indicated if post-MI with LVEF less than 40%, symptomatic HF or
DM, and receiving ACE inhibitors; however, contraindicated if creatinine clearance (CrCl) less than 30
mL/minute or serum potassium more than 5 mEq/L
4. Lipid management High-intensity statins are indicated with a goal of less than 70 mg/dL
5. Discontinue NSAIDs except for ASA at time of presentation. Selective COX-2 inhibitors and other
nonselective NSAIDs have been associated with increased cardiovascular death. Pain should be treated

ACCP Updates in Therapeutics 2013: The Pharmacotherapy Preparatory Review and Recertification Course
2-218

Cardiology I

in a stepwise fashion, starting with acetaminophen or ASA. Small doses of narcotics or nonacetylated
salicylates are preferred to NSAIDs. Nonselective NSAIDs (i.e., naproxen) are last line if other pain control
measures are unsuccessful. In all cases, the lowest effective dose should be used for the shortest time.
6. Other goals Hemoglobin A1c less than 7%, smoking cessation, body mass index 18.524.9 kg/m2,
exercise at least five times/week
7. In patients requiring warfarin therapy with dual antiplatelet therapy (DAPT), an international normalized
ratio (INR) goal of 2.02.5 is recommended with low-dose ASA according to STEMI guidelines (class IIa,
LOE [level of evidence]: C)

Patient Cases
1. A 66-year-old, 70-kg woman with a history of MI, HTN, hyperlipidemia, and DM presents with suddenonset diaphoresis, nausea, vomiting, and dyspnea, followed by a bandlike upper chest pain (8/10) radiating
to her left arm. She had felt well until 1 month ago, when she noticed her typical angina was occurring with
less exertion. Electrocardiography showed ST-segment depression in leads II, III, and aVF and hyperdynamic T waves and positive cardiac enzymes. Home medications are ASA 81 mg/day, simvastatin 40 mg
every night, metoprolol 50 mg two times/day, and metformin 1 g two times/day. Which regimen is the best
treatment strategy for this patient?
A. ASA 325 mg and clopidogrel 600 mg 1; then 75 mg once daily, UFH 60-unit/kg bolus; then 12 units/
kg/hour titrated to 5070 seconds immediately plus eptifibatide 180-mcg/kg bolus 2; then 2 mcg/kg/
minute at the time of PCI.
B. ASA 325 mg and enoxaparin 70 mg subcutaneously two times/day plus cardiac catheterization for
possible PCI.
C. Medical management with abciximab 0.25-mg/kg bolus; then 0.125 mg/kg/minute for 12 hours plus
enoxaparin 80 mg subcutaneously two times/day, ASA 325 mg/day, and clopidogrel 300 mg 1; then 75
mg once a day.
D. Medical management with ASA 325 mg and clopidogrel 300 mg 1; then 75 mg once a day plus UFH
70-unit/kg bolus; then 15 units/kg/hour.
2. A 45-year-old patient received an elective percutaneous transluminal coronary angioplasty and DES in her
right coronary artery. Which duration best represents the minimum time DAPT should be continued?
A.
B.
C.
D.

At least 1 month.
At least 3 months.
At least 6 months.
At least 12 months.

ACCP Updates in Therapeutics 2013: The Pharmacotherapy Preparatory Review and Recertification Course
2-219

Cardiology I

Patient Cases (continued)


3. A 52-year-old man (weight 100 kg) with a history of HTN and hypertriglyceridemia presents to a major
university teaching hospital with a cardiac catheterization laboratory. He has 3 hours of crushing 10/10
substernal chest pain radiating to both arms that began while he was eating his lunch (seated), which is accompanied by nausea, diaphoresis, and shortness of breath. He has never before experienced chest pain of
this character or intensity. He usually can walk several miles without difficulty and is a 1 pack/day smoker.
Home medications are lisinopril 2.5 mg/day and gemfibrozil 600 mg two times/day. Current vital signs
include HR 68 beats/minute and BP 178/94 mm Hg. Electrocardiography shows a 3-mm ST-segment elevation in leads V2V4, I, and aVL. Serum chemistry values are within normal limits. The first set of cardiac
markers shows positive myoglobin concentrations, creatine kinase (CK) 175 units/L, myocardial band (MB)
17.4 units/L, and troponin T 0.8 mcg/L (normal defined as less than 0.1 mcg/L). Which regimen is best to
treat this patients STEMI?
A. Cardiac catheterization with primary PCI (stent) of occluded artery, together with abciximab 0.25mcg/kg intravenous push; then 0.125 mg/kg/minute for 12 hours, clopidogrel, ASA, and UFH.
B. Reteplase 10-unit bolus 2, 30 minutes apart, plus UFH 60-unit/kg bolus and a 12-unit/kg/hour
infusion, clopidogrel, and ASA.
C. Abciximab 0.25-mg/kg intravenous push and 0.125 mg/kg/minute for 12 hours plus enoxaparin 100
mg subcutaneously two times/day plus tenecteplase 25-mg intravenous push 1, ASA, clopidogrel,
and UFH.
D. Tirofiban 0.04 mcg/kg/minute 30 minutes; then 0.01 mcg/kg/minute plus UFH 60-unit/kg bolus and
a 12-unit/kg/hour infusion, clopidogrel, and ASA.
4.

A 75-year-old (weight 65 kg) male smoker has a history of HTN, benign prostatic hypertrophy, and lower
back pain. Three weeks ago, he began to experience substernal chest pain with exertion (together with
dyspnea), which radiated to both arms and was associated with nausea and diaphoresis. Episodes have increased in frequency to four or five times/day; they are relieved with rest. He has never had an ECG. Today,
he awoke with 7/10 chest pain and went to the emergency department of a rural community hospital 2 hours
later. He was acutely dyspneic and had ongoing pain. Home medications are ASA 81 mg/day for 2 months,
doxazosin 2 mg/day, and ibuprofen 800 mg three times/day. Vital signs include HR 42 beats/minute (sinus
bradycardia), BP 104/48 mm Hg, and weight 61 kg. Laboratory results include blood urea nitrogen (BUN)
45 mg/dL, SCr 2.5 mg/dL, CK 277 units/L, creatine kinase myocardial band (CK-MB) 35.2 units/L, and
troponin T 1.5 mcg/L (less than 0.1 mcg/L). His ECG shows a 3-mm ST-segment elevation. Aspirin, clopidogrel, and sublingual nitroglycerin were given in the emergency department. Which regimen is best to
recommend?
A. Alteplase 15 units intravenously plus enoxaparin 30 mg intravenous bolus.
B. UFH 4000 unit intravenous bolus followed by 800 units intravenously per hour.
C. Tenecteplase 35 mg intravenously plus UFH 4000-unit intravenous bolus followed by 800 units
intravenously per hour.
D. Diagnostic cardiac catheterization for possible primary PCI.

ACCP Updates in Therapeutics 2013: The Pharmacotherapy Preparatory Review and Recertification Course
2-220

Cardiology I

II. ACUTE DECOMPENSATED HEART FAILURE (ADHF)


A. Hemodynamic Parameters
Table 13. Hemodynamic Values in Patients with ADHF and Sepsis
Parameter
Mean arterial pressure (MAP) (mm Hg)
Heart rate (HR) (beats/minute)
Cardiac output (CO) (L/minute)
Cardiac index (CI) (L/minute/m2)
Pulmonary capillary wedge pressure (PCWP) (mm Hg)
Systemic vascular resistance (SVR) (dynessecondcm-5)
Central venous pressure (CVP) (mm Hg)

Normal
Value
80100
6080
47
2.83.6
812a
8001200
26

Typical
ADHF Value
6080
7090
24
1.32
1830
15003000
615

Typical
Sepsis Value
6080
90100
58
3.54
58
300800
26

1518 mm Hg is often desired/optimal in patients with HF to ensure optimal filling pressures.


ADHF = acute decompensated heart failure; CI = CO/BSA (BSA = body surface area); HF = heart failure; MAP = diastolic blood pressure +
[1/3(systolic blood pressure diastolic blood pressure)]; SVR = [(MAP CVP)/CO] 80.
Blood Pressure (BP) = Cardiac Output (CO) x SVR
CO = Stroke Volume (SV) x HR
a

B. Clinical Presentation
Table 14. Signs and Symptoms of ADHF
Congestion (elevated PCWP)
Dyspnea on exertion or at rest
Orthopnea, paroxysmal nocturnal dyspnea
Peripheral edema
Rales
Early satiety, nausea/vomiting
Ascites
Hepatomegaly, splenomegaly
Jugular venous distention
Hepatojugular reflux

Hypoperfusion (reduced CO)


Fatigue
Altered mental status or sleepiness
Cold extremities
Worsening renal function
Narrow pulse pressure
Hypotension
Hyponatremia

ADHF = acute decompensated heart failure; CO = cardiac output; PCWP = pulmonary capillary wedge pressure.

ACCP Updates in Therapeutics 2013: The Pharmacotherapy Preparatory Review and Recertification Course
2-221

Cardiology I

Table 15. Forester Hemodynamic Subsets and Therapy


CI
2.2

Subset I.
Normal
Warm and dry

Subset II.
Congestion
Warm and wet

Subset III.
Hypoperfused
Cold and dry

Subset IV.
Congestion + hypoperfusion
Cold and wet

18
PCWP
Subset I Warm and Dry (Normal Parameters)
(PCWP 1518 mm Hga AND CI greater than 2.2 L/minute/m2)
Optimize oral medications
Subset II Warm and Wet (Pulmonary/Peripheral Congestion)
(PCWP greater than 18 mm Hg AND CI greater than 2.2 L/minute/m 2)
IV diuretics IV vasodilators (venousc)
Subset III Cold and Dry (Hypoperfusion)
(PCWP 1518 mm Hga AND CI less than 2.2 L/minute/m2)
If PCWP < 15 mm Hg, IVF until PCWP 1518 mm Hg
If PCWP 15 mm Hg and MAP < 50 mm Hg, IV dopamine
If PCWP 15 mm Hg, MAP 50 mm Hg and compelling reason for inotrope,b IV inotrope
If PCWP 15 mm Hg, MAP 50 mm Hg, and no compelling reason for inotrope,b IV vasodilator (arteriald)
Subset IV Cold and Wet (Pulmonary/Peripheral Congestion AND Hypoperfusion)
(PCWP greater than 18 mm Hg AND CI less than 2.2 L/minute/m 2)
IV diuretics +
If MAP < 50 mm Hg, IV dopamine
If MAP 50 mm Hg and compelling reason for inotrope,b IV inotrope
If MAP 50 mm Hg and no compelling reason for inotrope,b IV vasodilator (venous and/or arterialc,d)
Goal PCWP is 812 mm Hg in a normal patient and 1518 in a patient with heart failure. If PCWP is less than 15 mm Hg in a patient with heart
failure, either remove fluid restriction or cautiously administer fluids until PCWP is 1518 mm Hg and then reassess CI.
b
Compelling reason for inotrope = SBP < 90 mm Hg, symptomatic hypotension, or worsening renal function.
c
Venous vasodilator reduce PCWP.
d
Arterial vasodilator reduce SVR with compensatory increase in CI.
CI = cardiac index; IV = intravenous; IVF = intravenous fluid; MAP = mean arterial pressure; PCWP = pulmonary capillary wedge pressure.
a

C. Chronic HF Therapy in the Setting of Acute Decompensation


1. It is critical to initiate and/or continue standard HF therapies unless there is a compelling reason to
avoid or discontinue such therapies (e.g., hypotension, cardiogenic shock).
2. ACE inhibitors
a. Caution with initiation or up-titration during aggressive diuresis
b. Increases in SCr (decrease in glomerular filtration rate of 20% or more) from ACE inhibitor use
are not associated with worse outcomes.

ACCP Updates in Therapeutics 2013: The Pharmacotherapy Preparatory Review and Recertification Course
2-222

Cardiology I

3. -Blockers
a. Do not discontinue in patients who are stable on dose before admission (i.e., recent initiation or uptitration was not responsible for decompensation).
b. Do not initiate or up-titrate until euvolemic.
c. Hold if hemodynamically unstable.
4. Digoxin
a. Continue at dose to achieve serum digoxin concentration of 0.50.8 ng/mL.
b. Avoid discontinuation unless there is a compelling reason to do so because digoxin withdrawal
has been associated with worsening HF symptoms.
c. Caution if renal function begins to deteriorate or often fluctuates
D. ADHF Therapy Overview
Table 16. Overview of ADHF Guideline Recommendations
Diuretic Therapy
i. Recommended as intravenous loop diuretics for patients with fluid overload.
Change to oral route on day before discharge, if possible
ii. When response to diuretics is minimal, the following options should be considered:
(a) Fluid and sodium restriction,
(b) Initiation of increased doses or continuous infusion of loop diuretic,
(c) Addition of a second diuretic with a different MOA (metolazone, hydrochlorothiazide, chlorothiazide), or
(d) Ultrafiltration.
Inotropic Therapy
i. May be considered to relieve symptoms and improve end-organ function in patients with reduced LVEF and
diminished peripheral perfusion or end-organ dysfunction (low output syndrome), particularly if:
(a) Marginal systolic BP (< 90 mm Hg),
(b) Symptomatic hypotension despite adequate filling pressure, or
(c) No response to or intolerance of intravenous vasodilators.
ii. May be considered in similar patients with evidence of fluid overload if they respond poorly to intravenous
diuretics or manifest diminished or worsening renal function
Vasodilator Therapy
i. May be considered in addition to intravenous loop diuretics to rapidly improve symptoms in patients with
acute pulmonary edema or severe hypertension
ii. May be considered in patients with persistent symptoms despite aggressive diuretics and oral drug therapy
iii. When adjunctive therapy is required in addition to loop diuretics, intravenous vasodilators should be
considered over inotropic drugs
Invasive Hemodynamic Monitoringa
Routine use of hemodynamic monitoring with invasive intravenous lines (e.g., Swan-Ganz pulmonary artery
catheters) is not recommended
a

Several noninvasive methods for hemodynamic monitoring are currently available.

ADHF = acute decompensated heart failure; BP = blood pressure; LVEF = left ventricular ejection fraction; MOA = mechanism of action.

E. Diuretics are primarily used to medically manage patients with pulmonary and peripheral congestion or
wet (subset II or IV) HF.

ACCP Updates in Therapeutics 2013: The Pharmacotherapy Preparatory Review and Recertification Course
2-223

Cardiology I

Table 17. Diuretic Therapy for ADHF


Loop diuretics (ascending limb of loop of Henle)
i. Most widely used and most potent, effective at low CrCl (< 30 mL/minute)
ii. Furosemide most commonly used; furosemide 40 mg po = furosemide 20 mg IV = bumetanide 1 mg IV/po =
torsemide 20 mg IV/po
Thiazides (distal tubule)
i. Relatively weak diuretics when used alone, not effective at low glomerular filtration rate (CrCl < 30 mL/minute)
ii. Reserved for add-on therapy in patients refractory to loops
Diuretic resistance
i. Increase dose before increasing frequency of loop diuretic
(Note: Ceiling effect at about 160200 mg IV furosemide)
ii. Add a second diuretic with a different mechanism of action
(a) Hydrochlorothiazide 12.525 mg po daily, metolazone 2.55 mg po daily (30 minutes before loop diuretic
administration)
(b) Chlorothiazide 250500 mg IV daily; consider if gastrointestinal edema; generic is very expensive
Reserve for NPO or refractory to other alternatives
iii. Continuous infusion of loop diuretic Furosemide 0.1 mg/kg/hour IV doubled every 48 hours, maximum 0.4
mg/kg/hour
Adverse effects: Electrolyte depletion (potassium, magnesium), worsening renal function
ADHF = acute decompensated heart failure; CrCl = creatinine clearance; IV = intravenous(ly); NPO = nothing by mouth; po = orally.

F. Inotropic therapy is primarily used to manage hypoperfusion or cold (subset III or IV) HF. It is
important to confirm that patients in subset III have adequate filling pressures (i.e., PCWP 1518 mm Hg)
before administering inotropic therapy.
Table 18. Inotropic Therapy for ADHF
Dobutamine (Dobutrex)
Milrinone (Primacor)
Mechanism
1-Agonist: Stimulates AC to convert ATP to
PDE inhibitor: Inhibits cAMP breakdown in heart
of action
cAMP to CO; slight peripheral vasodilation
to CO and in vascular smooth muscle to SVR
Clinical effects
Positive inotropic, chronotropic,
Positive inotropic and lusitropic effects, no direct
lusitropic effects
chronotropic effects
Indication
ADHF Cold and wet (Forester subset IV) or cold and dry exacerbations (Forester III) (if
PCWP > 15 mm Hg)
Dosing
Start 2.55 mcg/kg/minute IV; may
50 mcg/kg IVB; then 0.375 mcg/kg/minute IV;
titrate to maximum 20 mcg/kg/minute
may titrate to maximum 0.75 mcg/kg/minute
Typical dose
5 mcg/kg/minute IV
No bolus, 0.10.375 mcg/kg/minute IV
Half-life
2 minutes
1 hour, prolonged to 23 hours if HF and/or
CrCl < 50 mL/minute
Elimination
Hepatically metabolized (inactive), renally
90% renal
eliminated
Adverse effects Proarrhythmia, tachycardia, hypokalemia,
Proarrhythmia, hypotension (avoid bolus),
myocardial ischemia, tachyphylaxis
tachycardia, < 1% thrombocytopenia, possible
(> 72 hours); possible increased mortality
increased mortality with long-term use
with long-term use
Other comments Consider if hypotensive
Consider if receiving a -blocker
AC = adenylate cyclase; ADHF = acute decompensated heart failure; ATP = adenosine triphosphate; cAMP = cyclic adenosine monophosphate;
CO = cardiac output; CrCl = creatinine clearance; HF = heart failure; IV = intravenous(ly); IVB = intravenous bolus; PCWP = pulmonary
capillary wedge pressure; PDE = phosphodiesterase; SVR = systemic vascular resistance.

ACCP Updates in Therapeutics 2013: The Pharmacotherapy Preparatory Review and Recertification Course
2-224

Cardiology I

G. Vasodilator therapy is primarily used to manage pulmonary congestion or wet (subset II or IV) HF.
Venous vasodilation results in a reduction in pulmonary capillary wedge pressure (PCWP) and acute relief
of shortness of breath while awaiting the onset of diuretic effects. Vasodilators with arterial vasodilating
properties (nitroprusside and nesiritide) may also be used as an alternative to inotropes in patients with
elevated systemic vascular resistance (SVR) and low cardiac output (CO).
Table 19. Vasodilator Therapy for ADHF
Sodium Nitroprusside
(Nipride)
Mechanism of Nitric oxideinduced
action
stimulation of GC to
convert GTP to cGMP

Nesiritide
(Natrecor)
Recombinant B-type
natriuretic peptide binds
to natriuretic peptide
receptor A to stimulate GC
and production of cGMP;
natriuretic mechanism
unknown
Hemodynamic effects:
PCWP and SVR, CI,
minimal changes in HR
Neurohormonal effects:
NE, ET-1, and aldosterone
Natriuretic effects at
supratherapeutic doses
Warm and wet ADHF,
alternative to inotropes in
cold and wet ADHF

Clinical
effects

Balanced arterial and


venous vasodilator

Indication

Warm and wet ADHF,


alternative to inotropes
in cold and wet ADHF,
hypertensive crises
0.30.5 mcg/kg/minute IV, 2 mcg/kg IVB, 0.01 mcg/kg/
by 0.5 mcg/kg/minute up to minute IV
3 mcg/kg/minute
0.51 mcg/kg/minute IV
0.01 mcg/kg/minute IV
May omit bolus if low SBP
< 10 minutes
20 minutes
Cyanide hepatically
Natriuretic peptide receptor C
metabolized, thiocyanate
(no renal/hepatic adjustment)
renally excreted
Hypotension or cyanide or
Primarily hypotension (up
thiocyanate toxicity
to 1 hour), tachycardia (less
than inotropes)

Dosing

Typical dose
Half-life
Elimination

Adverse
effects

IV Nitroglycerin
Combines with sulfhydryl
groups in vascular endothelium
to create S-nitrosothiol
compounds that mimic nitric
oxides stimulation of GC and
production of cGMP
Preferential venous vasodilator
> arterial vasodilator, arterial
vasodilation at high doses (e.g.,
100 mcg/minute)

Warm and wet ADHF, ACS, or


hypertensive crises

5 mcg/minute IV, by 5 mcg/


minute
up to 200 mcg/minute
2575 mcg/minute IV, titrated
to response
14 minutes
Inactive metabolites in urine

Hypotension, reflex tachycardia,


headache, tachyphylaxis

ACS = acute coronary syndromes; ADHF = acute decompensated heart failure; cGMP = cyclic guanine monophosphate; CI = cardiac index;
ET-1 = endothelin; GC = guanylate cyclase; GTP = guanosine triphosphate; HR = heart rate; IV = intravenous(ly); IVB = intravenous bolus;
NE = norepinephrine; PAC = pulmonary artery catheter; PCWP = pulmonary capillary wedge pressure; SBP = systolic blood pressure; SVR =
systemic vascular resistance.

ACCP Updates in Therapeutics 2013: The Pharmacotherapy Preparatory Review and Recertification Course
2-225

Cardiology I

Patient Cases
5. A 72-year-old man is admitted to the hospital for HF decompensation. The patient notes progressively increased dyspnea when walking (now 10 ft [3 m], previously 30 ft [6 m]) and orthopnea (now four pillows,
previously two pillows), increased bilateral lower extremity swelling (3+), 13-kg weight gain in the past
3 weeks, and dietary nonadherence. He has a history of idiopathic dilated cardiomyopathy (LVEF 25%,
NYHA class III), paroxysmal atrial fibrillation (AF), and hyperlipidemia. Pertinent laboratory values are
as follows: B-type natriuretic peptide (BNP) 2300 pg/mL (050 pg/mL), potassium (K+) 4.9 mEq/L, BUN
32 mg/dL, SCr 2.0 mg/dL (baseline 1.9 mg/dL), aspartate aminotransferase (AST) 40 IU/L, alanine aminotransferase (ALT) 42 IU/L, INR 1.3, aPTT 42 seconds, BP 108/62 mm Hg, HR 82 beats/minute, and O2
saturation 95%. Home drugs include carvedilol 12.5 mg two times/day, lisinopril 40 mg/day, furosemide 80
mg two times/day, spironolactone 25 mg/day, and digoxin 0.125 mg/day. Which regimen is best for treating
his ADHF?
A. Carvedilol 25 mg two times/day.
B. Nesiritide 2-mcg/kg bolus; then 0.01 mcg/kg/minute.
C. Furosemide 120 mg intravenously two times/day.
D. Milrinone 0.5 mcg/kg/minute.

6. After the initiation of intravenous loop diuretics with only minimal urine output, the patient is transferred to
the coronary care unit for further management of diuretic-refractory decompensated HF. His O2 saturation is
now 87% on 4-L nasal cannula, and an arterial blood gas is being obtained. His BP is 110/75 mm Hg, and his
HR is 75 beats/minute. The patients SCr and K+ concentrations have begun to rise; they are now 2.7 mg/dL
and 5.4 mmol/L, respectively. In addition to a one-time dose of intravenous chlorothiazide, which regimen is
most appropriate for this patient?
A.
B.
C.
D.
7.

Nitroglycerin 20 mcg/minute.
Sodium nitroprusside 0.3 mg/kg/minute.
Dobutamine 5 mcg/kg/minute.
Milrinone 0.5 mcg/kg/minute.

The patient initially responds with 2 L of urine output overnight, and his weight decreases by 1 kg the next
day. However, by day 5, his urine output has diminished again, and his SCr has risen to 4.3 mg/dL. He was
drowsy and confused this morning during rounds. His extremities are cool and cyanotic, BP is 89/58 mm Hg,
and HR is 98 beats/minute. It is believed that he is no longer responding to his current regimen. A SwanGanz catheter is placed to determine further management. Hemodynamic values are cardiac index (CI)
1.5 L/minute/m 2, SVR 2650 dynes/cm-5, and PCWP 30 mm Hg. Which regimen is most appropriate for his
current symptoms?
A.
B.
C.
D.

Milrinone 0.2 mcg/kg/minute.


Dobutamine 5 mcg/kg/minute.
Nesiritide 2-mcg/kg bolus; then 0.01 mcg/kg/minute.
Phenylephrine 20 mcg/minute.

ACCP Updates in Therapeutics 2013: The Pharmacotherapy Preparatory Review and Recertification Course
2-226

Cardiology I

III. ACUTE LIFE-THREATENING ARRHYTHMIAS


A. Adult Cardiac Arrest
Table 20. Select ACLS Algorithmsa
1. Start CPR (give oxygen; attach monitor/defibrillator)
2. Rhythm shockable? (if YES go to Pulseless VT/VF No. 3; if NO, go to Asystole/PEA No. 11)
Algorithm for Pulseless Ventricular Tachycardia or Fibrillation
3. DEFIBRILLATION (Shock #1)
4. CPR 2 minutes
Establish IV/IO access
5. Reassess rhythm Shock if appropriate and proceed
If no sign of ROSC, go to Asystole/PEA algorithm
If ROSC, initiate post-cardiac arrest care
6. DEFIBRILLATION (Shock #2)
7. CPR 2 minutes
Epinephrine 1 mg IV/IO every 35 minutes
Vasopressin 40 units IV/IO 1 (replaces first or second epinephrine dose)
Consider advanced airway, capnography
8. Reassess rhythm Shock if appropriate and proceed
If no sign of ROSC, go to Asystole/PEA algorithm
If ROSC, initiate post-cardiac arrest care
9. DEFIBRILLATION (Shock #3)
10. CPR 2 minutes
Amiodaroneb 300 mg IV/IO 1; may repeat at 150-mg bolus x 1
Reversible causes of the event should be identified and correctedc
Algorithm for Asystole or PEA
11. CPR 2 minutes
IV/IO access
Epinephrine 1 mg IV/IO every 35 minutes
Vasopressin 40 units IV/IO 1 (replaces first or second epinephrine dose)
Consider advanced airway, capnography
12. Reassess rhythm Shock if appropriate and proceed to No. 6 or 7 for pulseless VT/VF
If no sign of ROSC, proceed
13. CPR 2 minutes
Treat reversible causesb
14. Reassess rhythm Shock if appropriate and proceed to No. 6 or 7 for pulseless VT/VF
If no sign of ROSC, continue steps 1114
If ROSC, initiate post-cardiac arrest care
If no IV/IO access, endotracheal administration of epinephrine, lidocaine, and atropine is allowed at 22.5 times the recommended IV/IO dose.
Dilute this dose with 510 mL of sterile water or normal saline.
b
If amiodarone is unavailable, lidocaine may be considered. Lidocaine 11.5 mg/kg IV, repeat 0.50.75 mg/kg IV/IO every 510 minutes
(maximum 3 mg/kg). Lidocaine has not been shown to improve ROSC and hospital admission compared with amiodarone.
c
Hypovolemia, hypoxia, hydrogen ion (acidosis), hypo-/hyperkalemia, hypothermia, tension pneumothorax, tamponade (cardiac), toxins,
thrombosis (pulmonary), thrombosis (coronary).
ACLS = advanced cardiovascular life support; CPR = cardiopulmonary resuscitation; DCC = direct current conversion; HR = heart rate; IO =
intraosseous(ly); IV = intravenous(ly); PEA = pulseless electrical activity; ROSC = return of spontaneous circulation.
a

ACCP Updates in Therapeutics 2013: The Pharmacotherapy Preparatory Review and Recertification Course
2-227

Cardiology I

B. Symptomatic Bradycardia
1. If unstable, atropine 0.5 mg every 35 minutes (maximal dose 3 mg). (Note: Unstable = hypotension, acutely
altered mental status, signs of shock, ischemic chest discomfort, acute HF)
2. If atropine fails, transcutaneous pacing, dopamine 210 mcg/kg/minute, or epinephrine 210 mcg/minute
C. Symptomatic Tachycardia
1. If unstable, synchronized cardioversion
2. If stable, determine whether QRS complex is narrow or wide.
a. Narrow complex tachycardia (QRS less than 120 milliseconds) Usually atrial arrhythmias
i. Regular ventricular rhythm Supraventricular tachycardia (SVT) or sinus tachycardia likely
(a) Vagal maneuvers and/or adenosine 6-mg intravenous push, followed by a 20-mL saline
flush; then, a 12-mg intravenous push (may repeat once)
(1) Rapid push followed by elevation of arm to increase circulation
(2) Larger doses may be needed in patients taking theophylline, caffeine, or theobromine.
(3) Initial dose should be reduced to 3 mg in patients taking dipyridamole or carbamazepine
and in patients after heart transplantation, as well as when the drug is being given by
central access.
(4) Use adenosine cautiously in severe CAD.
(5) Adenosine should not be given to patients with asthma.
(6) Do not give adenosine for unstable or for irregular or polymorphic wide complex
tachycardias because it may cause degeneration to VF.
(b) If vagal maneuvers or adenosine fails to convert paroxysmal SVT, calcium channel blockers
or -blockers can be used. If WPW (Wolff-Parkinson-White) syndrome, avoid verapamil,
diltiazem, and digoxin
ii. Irregular (narrow complex) ventricular rhythm AF (or possibly atrial flutter)
(a) General management should focus on control of the rapid ventricular rate.
(1) Usually nondihydropyridine calcium channel blockers (diltiazem, verapamil) or
-blockers; digoxin is sometimes useful
(2) Rate is acceptable if it is less than 110 beats/minute at rest in asymptomatic persistent
AF.
(b) If hemodynamically unstable, synchronized cardioversion recommended
(c) Patients with AF for more than 48 hours are at high risk of cardioembolic events and should
not be immediately cardioverted if stable.
(d) TEE (transesophageal echocardiography) before cardioversion is an alternative strategy to
ensure the absence of left atrial clot.
(e) Risk of thromboembolic event surrounding cardioversion (both pharmacologic and electrical)
is greatest within the first 10 days.
(f) Cardioversion:
(1) If AF for up to 7 days, either elective direct current conversion (DCC) or chemical
cardioversion with:
(A) Flecainide, dofetilide, propafenone, ibutilide, or amiodarone (proven efficacy)
(B) Digoxin and sotalol NOT recommended and may be harmful
(C) Disopyramide, quinidine, and procainamide are less effective or incompletely
studied.
(2) If AF greater than 7 days, either elective DCC or chemical cardioversion with
dofetilide, amiodarone, or ibutilide (proven efficacy)
b. Wide complex tachycardia (QRS greater than 120 milliseconds) Usually ventricular arrhythmias
i. VT or unknown mechanism

ACCP Updates in Therapeutics 2013: The Pharmacotherapy Preparatory Review and Recertification Course
2-228

Cardiology I

(a) Consider adenosine only if regular and monomorphic.


(b) Intravenous procainamide, amiodarone (or sotalol); lidocaine second line
(c) Avoid procainamide and sotalol if prolonged QT.
ii. Definite SVT with aberrancy Likely transiently slowed or converted by adenosine
iii. Polymorphic (irregular) VT
(a) Induced primarily when the QTc interval is greater than 500 milliseconds (torsades de pointes)
(b) If unstable, polymorphic (irregular) VT requires immediate defibrillation with the same
strategy as VF.
(c) If stable, intravenous Mg2+ 1- to 2-g intravenous bolus (maximum 16 g/24 hours) may be
given; however, this is only supported by observational studies in the setting of wide QRS.
(d) Withdrawal of QT-prolonging medications, correction of low magnesium ion (Mg2+) or K+
(1) Class I and III antiarrhythmic drugs
(2) Assess for drug interactions by cytochrome P450 (CYP) 3A4 (e.g., azole antifungals,
erythromycin).
(3) Assess for other QT-prolonging drugs such as haloperidol, ziprasidone, droperidol,
sulfamethoxazole/trimethoprim, promethazine, and tricyclic amine antidepressants.
D. Antiarrhythmic Overview
Table 21. Vaughn-Williams Antiarrhythmic Drug Classes

Class/Ion Affected
Agents
Class I/Na+ channel
blockers
IA Disopyramide
(INTERMEDIATE) Quinidine,
procainamide
IB Lidocaine,
(FAST) mexiletine,
phenytoin
IC Flecainide
(SLOW) Propafenone
Class II
-Blockers

Metoprolol,
esmolol,
atenolol
Class III
Amiodarone,a
K+ channel blockers
dronedarone,a
sotalol,b
dofetilide,
ibutilide
Class IV
Diltiazem,
Ca2+ channel blockers verapamil
a

Physiologic Effect

Result on
Electrophysiologic
Parameters

Clinical Utility

Conduction velocity
Refractory period

QRS and QT

Atrial and ventricular


arrhythmias

Conduction velocity
Refractory period

QT

Ventricular
arrhythmias

Conduction velocity
Refractory period

QRS

Conduction velocity
Refractory period

HR and PR

Supraventricular
arrhythmias
and ventricular
arrhythmias
Atrial and ventricular
arrhythmias

Conduction velocity
Refractory period

QT

Atrial and ventricular


arrhythmias

Conduction velocity
Refractory period

HR and PR

Atrial and ventricular


arrhythmias

Amiodarone and dronedarone have Ib, II, and IV class activity in addition to class III actions.

Sotalol has 50%/50% -blocking properties/K+-blocking properties.


AV = atrioventricular; Ca2+ = calcium; HR = heart rate; K+ = potassium; Na+ = sodium; PR = time between the start of P wave and end of R wave on
an electrocardiograph; QRS = time between start of Q wave and end of S wave on an electrocardiograph; QT = time between start of Q wave and
end of T wave on an electrocardiograph.
b

ACCP Updates in Therapeutics 2013: The Pharmacotherapy Preparatory Review and Recertification Course
2-229

Cardiology I

Table 22. Antiarrhythmic Drug Properties and Dosing (class I and III agents only)
Drug
Quinidine
(Quinidex,
Quinaglute)

Procainamide
(Pronestyl)

Adverse Effects, Contraindications,


Pharmacokinetics, and Drug Interactions
Dosing by Indication
Class IA Na+ channel blockers
AEs: Nausea/vomiting/diarrhea (30%),
cinchonism (CNS and GI symptoms, tinnitus), strong
vagolytic and anticholinergic properties,
TdP (first 72 hours), hypotension, GI upset
PK: Half-life 59 hours
Potent inhibitor of CYP2D6; substrate and inhibitor of
CYP3A4
DIs: Warfarin, digoxin
AEs: Lupus-like syndrome
(30% if > 6 months), hypotension
(IV use, 5%), TdP
CI: LVEF < 40%
PK: Active metabolite NAPA (class III effects) may
accumulate in renal dysfunction

AF conversion:
Avoid use because of GI AEs
AF and VT maintenance:
Sulfate: 200400 mg po every 6 hours
Gluconate (CR): 324 mg po every 812 hours
Decrease dose 25% if CrCl < 10 mL/minute
AF conversion:
1 g IV for 30 minutes; then 2 mg/minute
(1-hour efficacy 51%)
AF maintenance:
No oral agent available
VT conversion:
20 mg/minute IV until 17 mg/kg, arrhythmia
ceases, hypotension, or QRS widens > 50%
VT maintenance:
14 mg/minute

Reduce dose in renal and liver dysfunction


AF conversion:
IR 200 mg (if < 50 kg) or 300 mg (if > 50 kg)
po every 6 hours
AF maintenance:
400800 mg/day in divided doses
(Recommended adult dose 600 mg/day given as
IR 150 mg po every 6 hours or as CR 300 mg po
every 12 hours)
If < 50 kg, moderate renal dysfunction (CrCl >
40 mL/minute) or hepatic dysfunction, maximum
400 mg/day
If severe renal dysfunction (IR only avoid CR)
CrCl 3040 mL/minute 100 mg every 8 hours
CrCl 1530 mL/minute every 12 hours
CrCl < 15 mL/minute every 24 hours
Ventricular tachycardias:
Use of VT has fallen out of favor because of the
availability of newer agents with less toxicity
+
Class IB Na channel blockers

Disopyramide AEs: Anticholinergic effects, TdP, ADHF (potent


negative inotropic effect)
(Norpace,
Norpace CR) CIs: Cardiogenic shock, congenital long QT syndrome,
second- or third-degree AV block, glaucoma
PK: Half-life 48 hours
Substrate of CYP2D6
DI: May enhance the effect of -blockers

Lidocaine
(Xylocaine)

AEs: CNS (perioral numbness, seizures, confusion,


blurry vision, tinnitus)
CI: Third-degree AV heart block
PK: Reduce dose in those with HF, liver disease, low
body weight and renal dysfunction and in the elderly
DI: Amiodarone (increased lidocaine levels)

Pulseless VT/VF conversion or VT with a pulse:


11.5 mg/kg IVP; repeat 0.50.75 mg/kg every
35 minutes (maximum 3 mg/kg)
(If LVEF < 40%: 0.50.75 mg/kg IVP)
(Amiodarone DOC in pulseless VT/VT
lidocaine acceptable if amiodarone not available)
VT maintenance: 14 mg/minute
Reduce maintenance infusion in liver disease

ACCP Updates in Therapeutics 2013: The Pharmacotherapy Preparatory Review and Recertification Course
2-230

Cardiology I

Table 22. Antiarrhythmic Drug Properties and Dosing (class I and III agents only) (continued)
Drug
Mexiletine
(Mexitil)

Propafenonea
(Rythmol,
Rythmol SR)

Flecainidea
(Tambocor)

Amiodarone
(Cordarone)

Adverse Effects, Contraindications,


Pharmacokinetics, and Drug Interactions
Dosing by Indication
AEs: CNS (tremor, dizziness, ataxia, nystagmus)
VT maintenance: 200300 mg po every 8 hours;
CI: Third-degree AV heart block
maximum 1200 mg/day
PK: Half-life 1220 hours
Substrate CYP2D6, CYP1A2
Reduce dose by 25%25% in hepatic impairment
Inhibitor CYP1A2
Class IC Na+ channel blockers (Note: Avoid in patients with HF or post-MI
Increased risk of sudden death)
AEs: Metallic taste, dizziness, ADHF, bronchospasm,
AF conversion:
bradycardia, heart block
600 mg po 1 (efficacy 45% at 3 hours)
(negative inotropy and -blocking properties)
450 mg po x 1 (weight < 70 kg)
CIs: HF (NYHA IIIIV), liver disease,
AF maintenance:
valvular disease (TdP), CAD, MI
HCl: 150300 mg po every 812 hours
PK: Half-life 1025 hours
HCl (SR): 225425 mg po every 12 hours
Substrate CYP2D6, CYP1A2, CYP3A4
Inhibitor CYP1A2, CYP2D6
Reduce dose 70%80% in hepatic impairment
DIs: Digoxin by 70%; warfarin by 50% as well as
drugs that inhibit CYP 2D6, 1A2, 3A4 (increased
propafenone)
AEs: Dizziness, tremor, ADHF
AF conversion: 300 mg po 1
(negative inotropy), vagolytic, anticholinergic,
(efficacy 50% at 3 hours)
hypotension
AF maintenance: 50150 mg po bid
CIs: HF, CAD, valvular disease, LVH (TdP)
PK: Half-life 1020 hours
Reduce dose by 50% when GFR < 50 mL/minute
Substrate CYP2D6, CYP1A2
Inhibitor CYP2D6
DI: Digoxin by 25%
Class III K+ channel blockers
AEs: Pulmonary fibrosis 3%17%, hyperthyroidism
3%, hypothyroidism 30%, neurologic toxicity 20%
40%, GI upset, photosensitivity, corneal deposits,
hepatitis, blue-gray skin 15%, TdP < 1%, heart block
14%, hypotension (IV), phlebitis (IV) (Ca2+and -blocking properties), bradycardia (IV)
CIs: Iodine hypersensitivity, hyperthyroidism, thirddegree AV heart block
PK: Half-life 58 days (average)
Inhibits CYP3A4/2D6/2C9/1A2/2C19 and intestinal
P-gp)
Substrate CYP3A4/1A2/2C19/2D6
DIs: Warfarin, digoxin, HMG-CoA reductase inhibitors
(maximal simvastatin dose 10 mg/day), phenytoin
50%, lidocaine,
and others
* Does not increase mortality in patients
with HF

AF conversion:
IV: 57 mg/kg IV over 3060 minutes; then
1.21.8 g/day continuous IV or divided oral
doses until 10 g
po: 1.21.8 g/day in divided doses until 10 g
AF maintenance:
200400 mg/day po
Pulseless VT/VF conversion:
300 mg or 5 mg/kg IVB in 20 mL of D5W or NS;
repeat 150 mg IVB every 35 minutes
Stable VT: 150 mg IVB in 100 mL of D5W for
10 minutes
VT/VF maintenance:
1 mg/minute 6 hours; then 0.5 mg/minute
(maximum 2.2 g/day)

ACCP Updates in Therapeutics 2013: The Pharmacotherapy Preparatory Review and Recertification Course
2-231

Cardiology I

Table 22. Antiarrhythmic Drug Properties and Dosing (class I and III agents only) (continued)
Drug
Sotalol

Dofetilide
(Tikosyn)

Ibutilide
(Covert)

Adverse Effects, Contraindications,


Pharmacokinetics, and Drug Interactions
AEs: ADHF, bradycardia, AV block, wheezing, 3%8%
TdP within 3 days of initiation, bronchospasm
(-blocking effects)
CIs: Baseline QT > 440 milliseconds or
CrCl < 40 mL/minute (AF only), LVEF <40%
PK: Renally eliminated, half-life 3040 hours
*Hospitalization ideal for initiation of therapy because
BW: Do not initiate if baseline QTc interval is >
450 milliseconds. If QTc exceeds 500 milliseconds
during therapy, reduce the dose, prolong the infusion
duration (IV product not available in the United
States), or discontinue use
Not effective for AF conversion
AEs: TdP (0.8%; 4% if no renal adjustment), diarrhea
CIs: Baseline QTc > 440 milliseconds or
CrCl < 20 mL/minute
PK: Renal and hepatic elimination
Half-life 610 hours
Substrate CYP3A4
DIs: CYP3A4 inhibitors and drugs secreted
by kidney (cimetidine, ketoconazole,
verapamil, trimethoprim, prochlorperazine,
megestrol), HCTZ
BW: Hospitalization mandatory for initiation, obtain
QTc 23 hours after each of the first five doses,
reduce 50% if QTc > 15%; NTE QTc > 500
milliseconds
* Does not increase mortality in patients with HF
AEs: TdP 8%, heart block (-blocking properties)
CIs: Baseline QTc > 440 milliseconds, LVEF < 30%,
concomitant antiarrhythmic drugs
PK: Half-life 212 hours (average 6)
DIs: CYP3A4 inhibitors or QT-prolonging drugs
* ECG monitoring during and 4 hours after CV

Dosing by Indication
AF maintenance (based on CrCl):
80 mg po bid (> 60 mL/minute)
80 mg po qd (4060 mL/minute)
Contraindicated < 40 mL/minute
VT maintenance (based on CrCl):
80 mg po bid (> 60 mL/minute)
80 mg po bid (3060 mL/minute)
80 mg po bid (1030 mL/minute)
80 mg po > qod (< 10 mL/minute)

AF conversion (based on CrCl):


(efficacy 12% at 1 month):
500 mcg po bid (> 60 mL/minute)
250 mcg po bid (4060 mL/minute)
125 mcg po bid (2040 mL/minute)
Contraindicated < 20 mL/minute
AF maintenance:
Titrate down based on QTc NTE 500 milliseconds
or > 15% in QTc

AF conversion:
1 mg IV ( 60 kg)
or 0.01 mg/kg IV (< 60 kg);
repeat in 10 minutes if ineffective
(efficacy 47% at 90 minutes)
BW: Potentially fatal arrhythmias (e.g.,
polymorphic VT) can occur with ibutilide,
usually in association with TdP; patients with
chronic AF may not be the best candidates for
ibutilide conversion

ACCP Updates in Therapeutics 2013: The Pharmacotherapy Preparatory Review and Recertification Course
2-232

Cardiology I

Table 22. Antiarrhythmic Drug Properties and Dosing (class I and III agents only) (continued)
Drug
Dronedarone
(Multaq)

Adverse Effects, Contraindications,


Pharmacokinetics, and Drug Interactions
AEs: Worsening HF, QT prolongation, hypokalemia or
hypomagnesemia with K+-sparing diuretics, hepatic
failure
CIs: QTc 500 milliseconds or PR 280 milliseconds,
NYHA class IV HF or NYHA class IIIII HF with
recent ADHF, severe hepatic impairment, second- or
third-degree AVB or HR < 50 beats/minute
PK: Half-life 1319 hours
Substrate 3A4
Inhibitor intestinal P-gp
DIs: CYP3A4 inhibitors, QT-prolonging drugs, simvastatin,
tacrolimus/sirolimus, warfarin, and other CYP3A4
substrates with narrow therapeutic range, digoxin and
other P-gp substrates (dabigatran, rivaroxaban)

Dosing by Indication
AF maintenance:
400 mg orally two times/day
Discontinue if QTc is 500 milliseconds or greater
BW: The risk of death is doubled when used
in patients with symptomatic HF with recent
decompensation requiring hospitalization
or NYHA class IV symptoms; use is
contraindicated in these patients
Use in patients with permanent AF doubles the
risk of death, stroke, and hospitalization for HF.
Use is contraindicated in patients with AF who
will not or cannot be converted to normal sinus
rhythm

Indicates pill-in-pocket approach can be used for selected patients.


ADHF = acute decompensated heart failure; AE = adverse effect; AF = atrial fibrillation; AV = atrioventricular; AVB = atrioventricular block;
bid = twice daily; BW = boxed warning; Ca2+ = calcium; CAD = coronary artery disease; CI = contraindication; CNS = central nervous system;
CR = controlled release; CrCl = creatinine clearance; CV = cardioversion; CYP = cytochrome P450; D5W = dextrose 5%; DI = drug interaction;
DOC = drug of choice; ECG = electrocardiogram; GFR = glomerular filtration rate; GI = gastrointestinal; HCl = hydrochloride; HCTZ =
hydrochlorothiazide; HF = heart failure; HMG-CoA = 3-hydroxy-3-methylglutaryl coenzyme A; HR = heart rate; IR = immediate release; IV
= intravenous; IVB = intravenous bolus; IVP = intravenous push; K+ = potassium; LVEF = left ventricular ejection fraction; MI = myocardial
infarction; MOA = mechanism of action; Na+ = sodium; NAPA = N-acetylprocainamide; NS = normal saline; NTE = not to exceed; NYHA =
New York Heart Association; P-gp = P-glycoprotein; PK = pharmacokinetics; po = orally; qd = once daily; qod = once every other day; QTc =
corrected QT interval; TdP = torsades de pointes; VF = ventricular fibrillation; VT = ventricular tachycardia.
a

E. Implantable Cardioverter Defibrillators


1. For primary prevention of SCD
a. Previous MI, at least 40 days prior and EF of 35% or less
b. Nonischemic dilated cardiomyopathy, LVEF of 35% or less receiving optimal chronic medications
for at least 3 months
c. Syncope with structural heart disease and inducible VT/VF during electrophysiologic study
d. High risk of life-threatening VT/VF; congenital long QT syndrome with recurrent symptoms or
torsades while receiving -blocker
e. Must have reasonable survival expectation for more than 1 year
2. For secondary prevention of SCD
a. Previous episode of resuscitated VT/VF, hemodynamically unstable VT with no completely
reversible cause, or sustained VT in presence of heart disease
b. Must be receiving optimal chronic medications (-blockers, ACE inhibitors)
c. Must have reasonable survival expectation for more than 1 year
F. Chronic Drug Therapy for Ventricular Arrhythmias
1. - blockers
a. Considered mainstay therapy
b. Effective in suppressing ventricular ectopic beats and reduce SCD in a spectrum of cardiac
disorders in patients with and without HF (nonsustained VT)
2. Amiodarone
a. No better than ICD in reducing SCD as a lone agent; no mortality benefit
b. Can be used to treat symptomatic nonsustained VT if -blockers not effective where ICD not
indicated
ACCP Updates in Therapeutics 2013: The Pharmacotherapy Preparatory Review and Recertification Course
2-233

Cardiology I

c. Can be used in combination with -blockers to decrease firing of ICD (defibrillator storm)
3. Sotalol
a. No mortality advantage
b. Can suppress VT and be used to decrease frequency of ICD firing
c. Greater proarrhythmic potential; avoid in patients with severely depressed LVEF or significant
HF; renal dosing required
Table 23. Alteration of Defibrillation Threshold
Threshold Alteration
Increase threshold
Decrease threshold

Medications
Amiodarone, lidocaine, and
mexiletine
Sotalol

Comments
Reprogram ICD, increased energy
(joules) required
May decrease energy needed for DCC

DCC = direct current conversion; ICD = implantable cardioverter defibrillator.

F. Treatment of Arrhythmias in Special Patient Populations


1. Heart failure
a. Avoid IA and IC agents.
b. Amiodarone and dofetilide (used atrial arrhythmias only) have a neutral effect on mortality in
patients with LV dysfunction post-MI.
c. Dronedarone (used in atrial arrhythmias only) is contraindicated in patients with symptomatic HF
with recent decompensation requiring hospitalization or NYHA class IV symptoms; risk of death
was doubled in these patients.
2. Acute MI
a. Avoid IA and IC agents.
b. CAST trial with class Ic agents (encainide, flecainide) showed increased mortality when used to
treat post-MI nonlife-threatening ventricular arrhythmias; avoid class Ic agents in patients with
structural heart disease.
c. Class IA medications Increased mortality in post-MI survivors
d. Amiodarone and dofetilide (used atrial arrhythmias only) have a neutral effect on mortality in
patients with LV dysfunction post-MI.
G. Drug-Induced Arrhythmias
1. Review all potential drug etiologies and treat appropriately.
a. Drug-induced QT prolongation
i. Ensure proper renal/hepatic dosing adjustments.
ii. Review electrolyte abnormalities and thyroid function tests.
iii. Ensure that all electrolytes are maintained at critical levels: K+ greater than 4 mmol/L and less
than 5 mmol/L, Mg++ greater than 2 mg/dL
iv. Ensure that all ECG parameters are within normal limits (e.g., QT interval less than 500
milliseconds).
b. Bradycardia or atrioventricular block
i. -Blocker, calcium channel blocker, digoxin
ii. Administer antidote if appropriate (e.g., calcium for calcium channel blocker toxicity).
c. Review for drug interactions. Antiarrhythmic agents have drug-drug interactions that may cause
significant outcomes.

ACCP Updates in Therapeutics 2013: The Pharmacotherapy Preparatory Review and Recertification Course
2-234

Cardiology I

Patient Cases
8.

A 68-year-old man is admitted after an episode of syncope with a presyncopal syndrome of seeing black
spots and experiencing dizziness before passing out. Telemetry monitor showed sustained VT for 45 seconds. His medical history includes HF NYHA class III, LVEF 30%, MI 2, HTN 20 years, LV hypertrophy, diabetes mellitus, and diabetic nephropathy. His drugs include lisinopril 5 mg/day, furosemide 20 mg
two times/day, metoprolol 25 mg two times/day, digoxin 0.125 mg/day, glyburide 5 mg/day, and aspirin 81 mg/
day. His BP is 120/75 mm Hg, HR 80 beats/minute, BUN 30 mg/dL, and SCr 2.2 mg/dL. Which is the best
therapy to initiate for conversion of his sustained VT?
A.
B.
C.
D.

Amiodarone 150 mg intravenously for 10 minutes; then 1 mg/minute for 6 hours; then 0.5 mg/minute.
Sotalol 80 mg two times/day titrated to QTc of about 450 milliseconds.
Dofetilide 500 mcg two times/day titrated to QTc of about 450 milliseconds.
Procainamide 20 mg/minute, with a maximum of 17 mg/kg.

9. The patient presents to the emergency department 3 months after amiodarone maintenance initiation (he
refused ICD placement) after a syncopal episode during which he lost consciousness for 30 seconds, according to witnesses. He also has rapid HR episodes during which he feels dizzy and light-headed. He feels very
warm all the time (he wears shorts, even though it is winter), is unable to sleep, and has experienced a 3-kg
weight loss. He received a diagnosis of hyperthyroidism caused by amiodarone therapy. On telemetry, he
shows runs of nonsustained VT. Which duration would best predict the duration of amiodarone-associated
hyperthyroidism in this patient?
A. Never.
B. 1 month.
C. 6 months.
D. 1 year.
10. A 64-year-old woman presents to the emergency department with a chief concern of palpitations. Her medical history includes HTN controlled with a diuretic and inferior-wall MI 6 months ago. She is pale and
diaphoretic but able to respond to commands. The patients laboratory parameters are within normal limits.
Her vital signs include BP 95/70 mm Hg and HR 145 beats/minute; telemetry shows sustained VT. Although
initially unresponsive to -blockers, the patient is successfully treated with lidocaine. Subsequent electrophysiologic testing reveals inducible VT, and sotalol 80 mg orally twice daily is prescribed. Two hours after
the second dose, the patients QTc is 520 milliseconds. Which regimen change would be most appropriate
for this patient?
A.
B.
C.
D.

Continue sotalol at 80 mg orally twice daily.


Increase sotalol to 120 mg orally twice daily.
Discontinue sotalol and initiate dofetilide 125 mcg orally twice daily.
Discontinue sotalol and initiate amiodarone 400 mg orally three times/day.

ACCP Updates in Therapeutics 2013: The Pharmacotherapy Preparatory Review and Recertification Course
2-235

Cardiology I

IV. HYPERTENSIVE CRISES


A. Definitions

Hypertensive Emergency
Severe elevations in BP (usually greater than
180/110 mm Hg) with the presence of acute
or ongoing target-organ damage

Requires immediate BP lowering (not


necessarily to normal ranges) to prevent or limit
further target-organ damage

Hypertensive Urgency
Accelerated, malignant, or perioperative
hypertension in the absence of symptoms
or new or progressive target-organ damage
Short-term risk is not as high, so BP reduction
occurs over several days, not immediately.

1. Look for acute target-organ damage (e.g., hypertensive encephalopathy, intracranial hemorrhage, or
other acute neurologic deficit; UA or acute MI; acute HF; pulmonary edema (shortness of breath);
aortic dissection; retinopathy or papilledema; decreased urine output or acute renal failure; eclampsia)
2. Presence of acute target-organ damage determines the treatment approach.
B. Goals
1. Hypertensive emergency Lower mean arterial pressure (MAP) by no more than 25% or diastolic BP
to 100110 mm Hg within 3060 minutes.
a. Intravenous medications used
b. No one drug of choice. Agents are chosen on the basis of patient characteristics (see Table 25).
c. Patients are usually admitted for intensive care unit (ICU) care and close follow-up.
2. Hypertensive urgency Short-term risk not as high as in emergency; oral medications can be used and
BP lowered over several days.
a. No proven benefit exists from rapid reduction in BP in patients with severe asymptomatic
hypertension.
b. The choice of agent used in this setting varies, and in many cases, adjusting chronic oral therapy
(increasing doses), reinitiating therapy in the nonadherent, or adding new agent (i.e., diuretic) to
chronic therapy is appropriate.
c. In some situations, shorter-acting agents may be chosen.
d. All patients with hypertensive urgency should be reevaluated within and no later than 7 days
(preferably after 13 days).

ACCP Updates in Therapeutics 2013: The Pharmacotherapy Preparatory Review and Recertification Course
2-236

Cardiology I

C. Treatment Options
Table 24. Commonly Used Intravenous Drugs for Hypertensive Emergencies
Drug (onset, duration)
Sodium nitroprusside
(Nipride)
(immediate, 23 minutes)

Intravenous Dose

Adverse Effects

Vasodilators
0.250.5 mcg/kg/minute,
maximum 3 mcg/kg/minute

Nitroglycerin
510 mcg/minute,
(25 minutes, 510 minutes) maximum 100 mcg/minute
Hydralazine
(Apresoline)
10 minutes, 14 hours)
Enalaprilat
(Vasotec)
(within 30 minutes, 1224
hours)
Fenoldopam
(Corlopam)
(< 5 minutes, 30 minutes)
Nicardipine
(Cardene)
(15 minutes, 1530
minutes Up to 4 hours if
prolonged infusion)
Clevidipine
(Cleviprex)
(24 minutes, 515 minutes)

510 mg every 46 hours


(not to exceed 20 mg/dose)
0.6251.25 mg every 46 hours,
maximum 5 mg every 6 hours

0.1 mcg/kg/minute,
maximum 1.6 mcg/kg/minute
515 mg/hour,
maximum 15 mg/hour

Cyanide/thiocyanate toxicity, nausea, vomiting,


methemoglobinemia
CIs: Renal, hepatic failure
Caution: Increased ICP
Headache, nausea, vomiting, tachyphylaxis,
methemoglobinemia
Caution: Increased ICP
Reflex tachycardia, headache, flushing
Caution: Angina/MI, increased ICP, aortic
dissection
Renal insufficiency/failure, hyperkalemia
CIs: Pregnancy, renal artery stenosis
(Note: Long half-life)
Headache, flushing, tachycardia, cerebral
ischemia
Caution: Glaucoma
Reflex tachycardia, nausea, vomiting,
headache, flushing
Caution: Angina/MI, acute HF, increased ICP

12 mg/hour
maximum 16 mg/hour

Patients with renal failure and hepatic failure


and geriatric patients not specifically studied
CIs: Soy/egg product allergy, severe aortic
stenosis, defective lipid metabolism
Caution: HF, concomitant -blocker use, reflex
tachycardia, rebound hypertension
Adrenergic Inhibitors
Esmolol
250400 mcg/kg/minute IVB;
Bronchospasm, HF exacerbation, bradycardia/
(Brevibloc)
then 50100 mcg/kg/minute
heart block
(12 minutes, 1030 minutes) infusion, maximum 300 mcg/
Caution: Acute HF, asthma, heart block
kg/minute
Labetalol
2080 mg every 15 minutes
Same as esmolol
(Normodyne, Trandate)
OR 0.52 mg/minute
(510 minutes, 36 hours)
maximum 300 mg/24 hours
CI = contraindication; HF = heart failure; ICP = intracranial pressure; IVB = intravenous bolus; LD = loading dose; MI = myocardial infarction.

ACCP Updates in Therapeutics 2013: The Pharmacotherapy Preparatory Review and Recertification Course
2-237

Cardiology I

Table 25. Agents Preferred for Hypertensive Crises Based on Comorbidities


Comorbidity
Acute aortic dissection

Preferred Intravenous Agent(s)


Labetalol, esmolol alone or in combination with nicardipine, clevidipine,
or nitroprusside
(-Blocker must precede other agents)
(Avoid hydralazine)
Acute heart failure
Nitroprusside, nitroglycerin, nesiritide, or ACE inhibitors in combination
with diuretics if pulmonary edema
(Note: Avoid -blockers)
Acute intracerebral hemorrhage/acute Labetalol, nicardipine
ischemic stroke
Acute myocardial infarction
-Blocker in combination with nitroglycerin
If heart rate < 70 beats/minute, consider nicardipine or clevidipine
Acute pulmonary edema
Nesiritide, nitroglycerin, nitroprusside
Acute renal failure
Fenoldopam, nicardipine, clevidipine
Eclampsia or preeclampsia
Hydralazine, labetalol, nicardipine
Hypertensive encephalopathy
Nitroprusside, labetalol, fenoldopam, nicardipine
Perioperative hypertension
Clevidipine, esmolol, nicardipine, nitroglycerin, nitroprusside
Sympathetic crisis
Nicardipine, fenoldopam, clevidipine, phentolamine
(Note: Avoid unopposed -blockade)
ACE = angiotensin-converting enzyme.

Table 26. Oral Short-Acting Antihypertensives


Drug

Oral Dose

Onset (minutes)

Adverse Effects

Captopril
(Capoten)
Clonidine
(Catapres)

6.550 mg

1530

Renal insufficiency/failure, hyperkalemia


CIs: Pregnancy, renal artery stenosis
Sedation, dry mouth, dizziness
Caution: Altered mental status
CIs: Severe carotid artery stenosis
Flushing, headache, edema
CIs: Severe aortic stenosis, coronary artery or
cerebrovascular disease
Bronchospasm, especially in patients with asthma
HF exacerbation; bradycardia/heart block
Caution: Acute HF

0.2 mg; then 0.1 mg/hour 515


up to 0.8 mg total

Nifedipine
1020 mg
(not
sublingual)
Labetalol
200400 mg repeated
(Normodyne, every 23 hours
Trandate)

1520

1530

CI = contraindication; HF = heart failure.

ACCP Updates in Therapeutics 2013: The Pharmacotherapy Preparatory Review and Recertification Course
2-238

Cardiology I

Patient Cases
11. A 68-year-old man, with a history of chronic kidney disease stage V on hemodialysis, HTN, CAD post-MI,
moderately depressed LVEF, and gastroesophageal reflux disease, presents with acute-onset shortness of
breath and chest pain. After his recent dialysis, he had a large barbecue meal with salt and smoked some
marijuana laced with cocaine. He was nonadherent to medical therapy for 2 days and noticed he had gained
2 kg in 24 hours. His baseline orthopnea worsened to sleeping sitting up in a chair for the 2 nights before
admission. He developed acute-onset chest tightness with diaphoresis and nausea, pain 7/10. He went to the
emergency department, where a BP of 250/120 mm Hg was noted. He had crackles halfway up his lungs on
examination, and chest radiography detected bilateral fluffy infiltrates with prominent vessel cephalization.
Electrocardiography showed sinus tachycardia HR 122 beats/minute and ST-segment depressions in leads
2, 3, and aVF. He was admitted for hypertensive emergency. Laboratory results are as follows: BUN 48 mg/
dL, SCr 11.4 mg/dL, BNP 2350 pg/mL, troponin T 1.5 mcg/L (less than 0.1 mcg/L), CK 227 units/L, and
CK-MB 22 units/L. Which medication is best to manage this patients hypertensive emergency?
A.
B.
C.
D.

Intravenous nitroglycerin 5 mcg/minute titrated to a 25% reduction in MAP.


Labetalol 2 mcg/minute titrated to a 50% reduction in MAP.
Sodium nitroprusside 0.25 mcg/kg/minute titrated to a 25% reduction in MAP.
Clonidine 0.1 mg orally every 2 hours as needed for a 50% reduction in MAP.

12. A 56-year-old white woman, with a long history of HTN because of nonadherence and recently diagnosed
HF (EF 35%), presents to the local emergency department with a BP 210/120 mm Hg and HR 105 beats/
minute. She states that she felt a little light-headed, but that now, she is feeling okay. She ran out of her BP
medications (including HCTZ, carvedilol, and lisinopril) 3 days ago. Current laboratory values are within
normal limits. Which medication is best to manage this patient?
A.
B.
C.
D.

Sodium nitroprusside 0.25 mcg/kg/minute titrated to a 25% reduction in MAP.


Labetalol 80 mg intravenously x 1; repeat until BP is less than 120/80 mm Hg.
Resumption of home medications; refer for follow-up within 2 days.
Resumption of home medications; initiate amlodipine 10 mg daily; refer for follow-up in 1 week.

V. PULMONARY ARTERIAL HYPERTENSION


A. Definition, Diagnosis, and Treatment Goals
1. Pulmonary arterial hypertension (PAH)
a. Idiopathic pulmonary arterial hypertension (IPAH)
i. Change in nomenclature during 2003 World Conference on Pulmonary Hypertension;
previously known as primary pulmonary hypertension
ii. Familial PAH
b. Secondary causes Scleroderma (most common), chronic thromboembolic disease, HIV (human
immunodeficiency virus), liver disease, connective tissue diseases, medications, toxins, others
2. Symptoms
a. Dyspnea with exertion (60% of patients), fatigue, chest pain, syncope, weakness (40%) Caused
by impaired oxygen delivery to tissues and diminished CO
b. Orthopnea, peripheral edema, liver congestion, abdominal bloating, and other signs of right
ventricular hypertrophy and failure occur when disease progresses to involve the heart.
3. Diagnosis and classification See Tables 27 and 28.

ACCP Updates in Therapeutics 2013: The Pharmacotherapy Preparatory Review and Recertification Course
2-239

Cardiology I

Table 27. Diagnostic Findings of PAH


Hemodynamic alterations
Electrocardiogram
Echocardiography
Chest radiography
Physical examination

mPAP > 25 mm Hg (> 30 mm Hg if exercising), PCWP 15 mm Hg, and PVR > 3


Wood units on RHC
Signs of RV hypertrophy, right-axis deviation, and anterior ST- and T-wave
abnormalities consistent with RV strain pattern
Estimated RV systolic pressure elevation, enlarged RV, RV dysfunction
Enlarged pulmonary arteries and diminished peripheral pulmonary vascular
markings, RV enlargement
Cool and/or cyanotic extremities, jugular venous distension, pulsatile hepatomegaly,
peripheral edema, ascites

mPAP = mean pulmonary arterial pressure; PA = pulmonary artery; PAH = pulmonary arterial hypertension; PCWP = pulmonary capillary
wedge pressure; PVR = pulmonary vascular resistance; RHC = right heart catheterization; RV = right ventricle/ventricular.

Table 28. World Health Organization Classification of Functional Status for PAH
Class
Definition
I
No symptoms (dyspnea, fatigue, syncope, chest pain) with normal daily activities
II
Symptoms with strenuous normal daily activities that slightly limit functional status and activity level
III
Symptoms of dyspnea, fatigue, syncope, and chest pain with normal daily activities that severely limit
functional status and activity level
IV
Symptoms at rest; cannot conduct normal daily activities without symptoms
PAH = pulmonary arterial hypertension.

4. Treatment goals
a. Relieve acute dyspnea symptoms.
b. Improve exercise capacity/quality of life and prevent death.
5. For acute vasodilator response testing:
a. Use intravenous epoprostenol, inhaled nitric oxide, or intravenous adenosine.
b. Positive response: Reduction in mean pulmonary arterial pressure (mPAP) of at least 10 mm Hg to
an absolute mPAP of less than 40 mm Hg
c. Positive response predicts mortality reduction with long-term calcium channel blocker or
vasodilator use.
B. Treatment of PAH
1. Reassessment should include functional class determination and 6-minute walk test every 36 months,
with right heart catheterization less often.
2. Satisfactory condition Functional class III, ambulated 380 m or greater (or 1250 ft) during 6-minute
walk test, with a CI of 2.2 L/minute/m2 or greater and a mPAP less than 12 mm Hg

ACCP Updates in Therapeutics 2013: The Pharmacotherapy Preparatory Review and Recertification Course
2-240

Cardiology I

Table 29. Initial PAH Treatment Algorithm


Supportive Care: Treat corrective causes of hypoxemia and
avoid dehydration, pain, fatigue, high altitude, smoking, pregnancy, iron deficiency, etc.
Oxygen to maintain O2 saturation > 90%, diuretic if peripheral edema or ascites
Oral anticoagulation, warfarin (INR 1.52.5) if IPAH diuretics digoxin
(anticoagulation to prevent catheter thrombosis [IV prostaglandin use] and venous thromboembolism)
Immunizations for influenza and Pneumococcus
Discuss effective methods of birth control with women of childbearing potential
Positive Response to Acute Vasoreactivity Testing
Initiate oral CCB
- If sustained response, continue CCB
- If no sustained response, see Lower Risk algorithm below
Negative Response to Acute Vasoreactivity Testing
a
Lower Risk
Higher Riskb
First line: ERAs or PDEIs (oral)
Epoprostenol or treprostinil (IV)
Alternatives: Epoprostenol or treprostinil (IV)
Iloprost (inhaled)
Iloprost (inhaled), treprostinil (SC)
ERAs or PDEIs (oral)
Treprostinil (SC)
Reassess: Consider combination therapy, investigational protocols, atrial septostomy, lung transplantation
Low risk (good prognosis) if no RV failure, gradual progression of symptoms, WHO class II or III, 6 MW more than 400 m, peak Vo2 more than
10.4 mL/kg/minute, minimal RV dysfunction, RAP less than 10 mm Hg, CI more than 2.5 L/minute/m 2, BNP minimally elevated.
a

High risk (poor prognosis) if RV failure, rapid progression of symptoms, WHO class IV, 6 MW less than 300 m, peak Vo2 < 10.4 mL/kg/minute,
substantial RV enlargement/dysfunction (or pericardial effusion or right atrial enlargement), RAP greater than 20 mm Hg, CI less than 2 L/
minute/m 2, BNP significantly elevated.
b

BNP = B-type natriuretic peptide; CCB = calcium channel blocker; CI = cardiac index; ERA = endothelin receptor antagonist; INR = international
normalized ratio; IPAH = idiopathic pulmonary arterial hypertension; IV = intravenous(ly); MW = minute walk; PDEI = phosphodiesterase
inhibitor; RAP = right atrial pressure; RV = right ventricular; SC = subcutaneous; WHO = World Health Organization.

Table 30. Overview of PAH Treatment Options


Drug/Mechanism/
Indication
Dose
Calcium channel
Varies by
blockers
agent and
patient
Class II PAH
tolerance

Epoprostenol
(Flolan, Veletri)
Prostanoid
Class IIIIV PAH

240 ng/kg/
minute IV

Adverse Effects
Hypotension, headache,
dizziness, peripheral
edema, cardiac
conduction delay
(diltiazem)

Jaw pain, nausea,


vomiting, flushing,
headache, muscle aches
and pain, catheterrelated thrombosis,
and IV line infections;
rebound worsening of
symptoms if abruptly
discontinued

Considerations
Should not be used empirically without positive
response to acute vasodilatory response testing.
Diltiazem, amlodipine, nifedipine most commonly
used
Select agent on the basis of HR at baseline
If tachycardic, choose diltiazem
If bradycardic, choose amlodipine, nifedipine
Continuous IV infusion by pump
Flolan: Unstable at acidic pH and room
temperature (refrigerate or use ice packs before
and during infusion)
Veletri: Stable at room temperature
Drug requires reconstitution in sterile environment
Medical emergency if infusion interrupted
(half-life 6 minutes) Spare drug cassette and
infusion pump should be kept available

ACCP Updates in Therapeutics 2013: The Pharmacotherapy Preparatory Review and Recertification Course
2-241

Cardiology I

Table 30. Overview of PAH Treatment Options (continued)


Drug/Mechanism/
Indication
Dose
Adverse Effects
Treprostinil
1.25- to
Severe erythema and
(Remodulin,
40-ng/kg/
induration (83%) and
Tyvaso)
minute SC
injection site pain
Prostanoid
infusion, IV (85%) limits use; also
Inhaled
headache, nausea,
Class IIIV PAH
diarrhea, rash

Inhaled iloprost
(Ventavis)
Prostanoid
Class IIIIV PAH

Bosentan
(Tracleer)
Nonselective
endothelin
receptor
antagonist
(ETA and ETB)
Class IIIIV PAH

Ambrisentan
(Letairis)
Selective
endothelin
receptor
antagonist
(ETA only)
Class IIIII PAH
Sildenafil
(Revatio)
Phosphodiesterase
inhibitor
Class IIIV PAH

Considerations
Longer half-life (t1/2 3 hours) Longer to seek
medical attention
Premixed, prefilled syringe easier to administer
Local treatments (hot/cold packs or topical
analgesics) can be used to minimize infusion site
discomfort
Moving infusion site every 3 days minimizes
irritation
Requires 69 inhalations daily (15 minutes each
with jet nebulizer)
Prodose AAD nebulization system required
Inhaled form has fewer systemic adverse reactions
Use no more than every 2 hours

2.5 1; then Mild, transient cough,


5 mcg/
flushing, headache,
inhalation
syncope
by nebulizer
six to nine
times/day
while awake
62.5125 mg Peripheral edema
Severe drug interactions with glyburide (increased
po twice
5%14%, hypotension
LFTs) and cyclosporine (decreased efficacy of
daily
7%, increased LFTs
both cyclosporine and bosentan)
11%, flushing 7%14%, Monitor LFTs monthly
palpitations 5%
Monitor hemoglobin/hematocrit every 3 months
Potential teratogen; if childbearing age, use two
contraceptive methods (reduced efficacy of
hormonal contraceptives); monthly pregnancy
test required
Efficacy decreased with inducers and toxicity
increased with inhibitors of CYP2C8/9 and 3A4
510 mg po
Peripheral edema
Caution with cyclosporine maximal dose is 5 mg
once daily
17%, hypotension
daily
0%, increased LFTs
Contraindicated in patients with idiopathic
0%2.8%, flushing 4%, pulmonary fibrosis
palpitations 5%, fluid
Potential teratogen (see above comments)
retention
Periodic Hgb/Hct monitoring
No CYP drug interactions documented

20 mg po
three times/
day

Headache, epistaxis,
facial flushing, bluish
or blurry vision, light
sensitivity, dyspepsia,
insomnia

Half-life 45 hours
May augment effects of other vasodilators when
used in combination (especially prostacyclin)
Contraindicated in patients receiving nitrates
Avoid combined use with strong CYP3A4
inhibitors (e.g., ritonavir, cimetidine,
erythromycin) and inducers (rifampin)

ACCP Updates in Therapeutics 2013: The Pharmacotherapy Preparatory Review and Recertification Course
2-242

Cardiology I

Table 30. Overview of PAH Treatment Options (continued)


Drug/Mechanism/
Indication
Dose
Adverse Effects
Tadalafil
40 mg po
Headache, flushing,
(Adcirca)
once daily
indigestion, nausea,
Phosphodiesterase
backache, myalgia,
inhibitor
nasopharyngitis,
respiratory tract
Class IIIV PAH
infection

Considerations
Half-life 17.5 hours
May augment effects of other vasodilators when
used in combination (especially prostacyclin)
Contraindicated in patients receiving nitrates
If CrCl 3180 mL/minute, initiate 20 mg po once
daily and titrate as tolerated
If CrCl < 30 mL/minute or hemodialysis, avoid use
If Child-Pugh class A or B, initiate 20 mg po once
daily and titrate as tolerated
If Child-Pugh class C, avoid use
Avoid use with potent CYP3A4 inhibitors/inducers

AAD = antiarrhythmic drug; CCBs = calcium channel blockers; CrCl = creatinine clearance; CYP = cytochrome P450; ET-1 antagonists =
endothelin-1 antagonists; Hct = hematocrit; Hgb= hemoglobin; HR = heart rate; IV = intravenous; LFT = liver function test; PAH = pulmonary
arterial hypertension; po = orally; SC = subcutaneous.

Patient Cases
13. A 38-year-old obese woman presents with increasing symptoms of fatigue and shortness of breath. She
could walk only 1020 ft at baseline and is now short of breath at rest. Her arterial blood gas is pH 7.31/Pco2
65/Po2 53/85% O2 saturation. She has three-pillow orthopnea and 3+ pitting edema in her lower extremities. Medical history is significant only for AF. Computerized tomographic angiography shows that her
pulmonary artery trunk is substantially enlarged, with a mean pressure of 56 mm Hg. Echocardiography
shows right atrial and ventricular hypertrophy. Chest radiography detects prominent interstitial markings.
Pertinent laboratory test values are BUN 21 mg/dL, SCr 1.2 mg/dL, AST 145 IU/L, ALT 90 IU/L, INR
2.1, and partial thromboplastin time 52 seconds; vital signs include BP 108/62 mm Hg and HR 62 beats/
minute. Home medications are warfarin 2.5 mg/day, ipratropium 2 puffs every 6 hours, salmeterol 2 puffs
two times/day, and diltiazem 480 mg/day. Her diagnosis is IPAH. Which regimen is the best evidence-based
management strategy?
A.
B.
C.
D.

Increase diltiazem to 600 mg/day.


Start sildenafil 20 mg three times/day.
Start epoprostenol 2 ng/kg/minute.
Start bosentan 62.5 mg two times/day.

14. A 48-year-old man with IPAH is admitted to the medical ICU for severe respiratory distress. Medications
before admission included bosentan and sildenafil. His vital signs include BP 97/45 mm Hg, HR 130 beats/
minute, and respiratory rate 24 breaths/minute, and his oxygen requirements are increasing. Recently, during a previous hospital admission, pulmonary artery catheter placement revealed an mPAP of 40 mm Hg,
right atrial pressure 16 mm Hg, CI 1.2 L/minute, and PCWP 15 mm Hg. Echocardiography reveals EF 60%
with significant right ventricular dilation. Which regimen is most appropriate?
A.
B.
C.
D.

Epoprostenol and add phenylephrine, if needed, for BP support.


Furosemide and add norepinephrine, if needed, for BP support.
Nitroprusside and add epinephrine, if needed, for BP support.
Dobutamine to increase CO.

ACCP Updates in Therapeutics 2013: The Pharmacotherapy Preparatory Review and Recertification Course
2-243

Cardiology I

REFERENCES
Acute Coronary Syndromes

Acute Arrhythmias/Advanced Cardiac Life Support

1. Jneid H, Anderson JL, Wright RS, et al. 2012


ACCF/AHA focused update of the guideline for
the management of patients with unstable angina/
non-ST-elevation myocardial infarction (updating
the 2007 guideline and replacing the 2011 focused
update): a report of the American College of Cardiology Foundation/American Heart Association
Task Force on Practice Guidelines. Circulation
2012;126:875-910.

1. Fuster V, Ryden LE, Cannom DS. ACC/AHA/


ESC 2006 guidelines for the management of patients with atrial fibrillation executive summary:
a report of the American College of Cardiology/
American Heart Association Task Force on Practice
Guidelines and the European Society of Cardiology
Committee for Practice Guidelines. J Am Coll Cardiol 2006;48:854-906.

2. OGara PT, Kushner FG, Ascheim DD, et al. 2013


ACCF/AHA guideline for the management of
ST-elevation myocardial infarction: a report of
the American College of Cardiology Foundation/
American Heart Association Task Force on Practice
Guidelines. J Am Coll Cardiol 2013;61:e78-e140.
3. Levine GN, Bates ER, Blankenship JC, et al. 2011
ACCF/AHA/SCAI guideline for percutaneous coronary intervention: a report of the American College of Cardiology Foundation/American Heart Association Task Force on Practice Guidelines and the
Society for Cardiovascular Angiography and Interventions. J Am Coll Cardiol 2011;58:e44-e122.
Acute Decompensated Heart Failure
1. Hunt SA, Abraham WT, Chin MH, et al. 2009 focused update incorporated into the ACC/AHA 2005
guidelines for the diagnosis and management of
heart failure in adults: a report of the American
College of Cardiology Foundation/American Heart
Association Task Force on Practice Guidelines
developed in collaboration with the International
Society for Heart and Lund Transplantation. J Am
Coll Cardiol 2009;53:1343-82.
2. Weintraub NL, Collins SP, Pang PS, et al. Acute
heart failure syndromes: emergency department
presentation, treatment, and disposition: current
approaches and future aims: a scientific statement
from the American Heart Association. Circulation
2010;122:1975-96.
3. Lindenfield J, Albert NM, Boehmer JP, et al. HFSA
2010 comprehensive heart failure practice guideline. J Card Fail 2010;16:e1-e156.

2. Wann LS, Curtis AB, January CT, et al. 2011


ACCF/AHA/HRS focused update on the management of patients with atrial fibrillation (updating the
2006 guideline): a report of the American College
of Cardiology Foundation/American Heart Association Task Force on Practice Guidelines. Circulation 2011;123:104-23.
3. Zipes DP, Camm AJ, Borggrefe M, et al. ACC/
AHA/ESC 2006 guidelines for the management of
patients with ventricular arrhythmias and the prevention of sudden cardiac death executive summary: a report of the American College of Cardiology/American Heart Association Task Force
and the European Society of Cardiology Committee for Practice Guidelines. J Am Coll Cardiol
2006;48:1064-108.
4. Blomstrm-Lundqvist C, Scheinman MM, Aliot
EM, et al. ACC/AHA/ESC 2003 guidelines for
the management of patients with supraventricular
arrhythmias executive summary: a report of the
American College of Cardiology/American Heart
Association Task Force on Practice Guidelines
and the European Society of Cardiology Committee for Practice Guidelines. J Am Coll Cardiol
2003;42:1493-531.
5. American Heart Association. 2010 guidelines for
cardiopulmonary resuscitation and emergency cardiovascular care. Circulation 2010;122:S729-S767.
Hypertensive Emergency
1. Rhoney D, Peacock WF. Intravenous therapy for
hypertensive emergencies, part 1. Am J Health Syst
Pharm 2009;66:1343-52.

ACCP Updates in Therapeutics 2013: The Pharmacotherapy Preparatory Review and Recertification Course
2-244

Cardiology I

2. Rhoney D, Peacock WF. Intravenous therapy for


hypertensive emergencies, part 2. Am J Health Syst
Pharm 2009;66:1448-57.
3. Saseen JJ, MacLaughlin EJ. Chapter 19. Hypertension.
In: Pharmacotherapy: A Pathophysiologic Approach,
8th ed. New York: McGraw-Hill, 2011:101-35.
4. Marik PE, Varon J. Hypertensive crises: challenges
and management. Chest 2007;131:1949-62.
5. Chobanian AV, Bakris GL, Black HR, et al. Seventh
report of the Joint National Committee on prevention, detection, evaluation, and treatment of high
blood pressure. Hypertension 2003;42:1206-52.
Pulmonary Artery Hypertension
1. McLaughlin VV, Archer SL, Badesch DB, et al.
ACCF/AHA 2009 expert consensus document
on pulmonary hypertension. J Am Coll Cardiol
2009;53:1573-619.
2. Badesch DB, Abman SH, Simonneau G, et al. Medical therapy for pulmonary arterial hypertension: updated ACCP evidence-based clinical practice guidelines. Chest 2007;131:1917-28.
3. McLaughlin VV, McGoon MD. Pulmonary arterial
hypertension. Circulation 2006;114:1417-31.

ACCP Updates in Therapeutics 2013: The Pharmacotherapy Preparatory Review and Recertification Course
2-245

Cardiology I

ANSWERS AND EXPLANATIONS TO PATIENT CASES


1. Answer: A
In this patient, the presence of ST-segment depression
on ECG, positive biomarkers for myocardial necrosis, at least three risk factors for CAD, and history of
CAD (prior MI) suggests a high risk of future events.
In such high-risk patients, cardiac catheterization (invasive strategy) is used to determine whether occluded
or partly occluded epicardial arteries exist, which ones
can be intervened on, and whether to make an intervention (stent or percutaneous transluminal coronary angioplasty). Medical management, as in Answer C and
Answer D, would not be preferred because of this patients risk category. Dual antiplatelet therapy is indicated for an early invasive strategy in the management
of an NSTEMI (Answer A). The GPIs can be useful in
high-risk features, typically those with positive cardiac
enzymes, when an anticoagulant strategy with heparin is
chosen regardless of whether the patient was adequately
pretreated with clopidogrel. Unfractionated heparin, after a 60-unit/kg bolus and 12-unit/kg/hour infusion, can
be titrated to an aPTT of 5070 seconds.
2. Answer: D
It is important to understand that this is in the context
of elective stent placement, not after ACS. In the nonACS setting, the duration of DAT is determined by the
type of stent placed (BMS vs. DES). After elective DES
placement, DAT is recommended for a minimum of 12
months because the risk of in-stent thrombosis is highest during this time. The recommendation is at least 1
monthand ideally up to 1 yearafter BMS placement
because endothelialization of the stent usually occurs
early, typically within 1 month after stenting. Bleeding
risk may be a reason to consider earlier termination (after a minimum of 1 month) of DAT after BMS.
3. Answer: A
Although this patient presented within 3 hours of chest
pain onset and is a thrombolytic candidate (within less
than 6 hours of onset is preferred), up to 95% of patients
can achieve normal, brisk TIMI-3 flow rates with primary PCI versus only 50%60% of patients achieving
normal TIMI-3 coronary flow with thrombolytic therapy. Because the patient presents to a hospital that can
perform a primary PCI with stent implantation, this is
the therapy of choice. Although GPIs have been studied

in combination with fibrinolytics and anticoagulants for


STEMI, the slight increase in TIMI-3 blood flow rates
was accompanied by a substantially increased risk of
bleeding; thus, they are not recommended. Tirofiban
and UFH alone are not indicated for the treatment of
STEMI (recommended for the medical management of
NSTEMI ACS only).
4. Answer: C
Unlike the patient in case 3, this patient presents with
a STEMI to a rural community hospital. He presents
within the window for thrombolytic therapy consideration (less than 6 hours after chest pain onset). He is experiencing complete heart block and bradycardia, which
could indicate an occlusion above the area perfusing
his sinoatrial and/or atrioventricular nodes. Because
he is still having ischemic chest pain and ST-segment
elevation, he should benefit from reperfusion therapy.
Enoxaparin (Answer A) is a treatment option for medical management with lytic therapy, but the patient is at
higher risk of bleeding from impaired enoxaparin clearance and requires a dosage adjustment. Furthermore,
he is at an age (older than 75) at which the intravenous
bolus should not be given, and the dosing of alteplase is
incomplete. Simply treating this patient conservatively
with UFH alone (Answer B) in the setting of ongoing
chest pain, shortness of breath, and pulmonary edema
is not optimal. Diagnostic catheterization and possible
PCI to determine whether an artery can be reperfused
(Answer D) may be desirable but is complicated because the patients SCr is elevated (2.5 mg/dL) and he
is in a rural hospital, which may lack the ability to assess him quickly enough (within a 90-minute goal). Because of the shorter half-life and ease of administration
of tenecteplase, it is preferable to alteplase. Clearance
of UFH is not as altered as enoxaparin and would be a
more appropriate therapy than enoxaparin in combination with a thrombolytic.
5. Answer: C
This patient, who has ADHF, is receiving a -blocker.
Although long-term -blockers can improve HF symptoms and reduce mortality, they can also worsen symptoms in the short term. It is recommended to keep the
maintenance -blocker therapy at the same or a slightly
reduced dose compared with the outpatient therapy in

ACCP Updates in Therapeutics 2013: The Pharmacotherapy Preparatory Review and Recertification Course
2-246

Cardiology I

patients with ADHF; increasing the -blocker dose before reaching euvolemia may acutely worsen his clinical picture. In patients admitted with volume overload
without substantial signs of reduced CO, it is reasonable
to try intravenous loop diuretics initially. As gut edema
increases, oral loop diuretics (notably furosemide) become less effective because of decreased absorption.
Nesiritide is a vasodilatory drug that can be initiated if
intravenous loop diuretic therapy fails, but because of
its adverse effects and substantial cost, it is not recommended before a trial of intravenous diuretics and other
potential therapies. Milrinone is an inotropic drug. Because of their adverse effects, inotropes are recommended in cold and wet exacerbations only after vasodilatory
medications have failed.
6. Answer: A
Intravenous vasodilators such as nitroglycerin and sodium nitroprusside are reasonable options if intravenous
diuretics fail and the patient progresses to acute pulmonary edema. Both agents rapidly cause venous vasodilation and reduce pulmonary filling pressures, which can
relieve acute shortness of breath. Nitroglycerin is the
optimal choice for this patient given the declining renal
function and concern about increased risk of thiocyanate
toxicity in this setting. Dobutamine is typically used in
states of low CO decompensation and is counteracted by
concomitant -blocker therapy, making it a poor choice
in patients receiving -blockers. Although milrinone is
a more acceptable inotropic agent in a patient receiving -blockers, the dosing strategy is inappropriate as an
initial dose. Finally, inotropes are generally reserved for
patients when other therapies have failed.
7. Answer: A
Signs of a decreased CO state in HF (e.g., increased
SCr, decreased mental status, cool extremities) suggest
a cold and wet state, and adjunctive therapy is indicated.
Positive inotropic agents such as milrinone will increase
CO to maintain perfusion to vital organs. Milrinone will
also vasodilate the peripheral vessels to unload the heart
(lower SVR). Again, although dobutamine would be a
potential choice in this patient, it is not recommended in
patients receiving -blockers. Although this patient has
low BP, his elevated SVR suggests that he will tolerate
the vasodilatory effects of milrinone. Although nesiritide would provide venous and arterial vasodilation, it is
relatively contraindicated in patients with a systolic BP

less than 100 mm Hg and absolutely contraindicated in


patients with a systolic BP less than 90 mm Hg. Phenylephrine has no positive beta effects, so it will not augment
contractility. In addition, it will cause vasoconstriction
through alpha stimulation, which will further increase
SVR and likely worsen CO. Vasoconstrictors are reserved
for patients in cardiogenic shock. Even though this patient shows signs of significant hypoperfusion, the BP is
not so low that it warrants vasopressor therapy.
8. Answer: A
Treatment options for sustained VT are dependent on
concomitant disease states, particularly LVEF (40%
cutoff). In a patient with LV dysfunction, class I agents
such as procainamide are contraindicated. In a patient
whose CrCl is less than 60 mL/minute, sotalol requires
a considerable dosage reduction to avoid excess torsades de pointes. Sotalol is not an effective cardioversion
drug but is more useful for preventing future episodes
of arrhythmias (maintaining sinus rhythm) once sinus
rhythm is achieved. Dofetilide is indicated only for AF,
not for ventricular arrhythmias; similarly, cardioversion
rates with dofetilide are low. Amiodarone is first-line
therapy for sustained VT in patients with severe renal
insufficiency, HF, and structural heart disease.
9. Answer: C
With the prolonged half-life of amiodarone and extensive fat tissue volume of distribution, it would be expected that hyperthyroid adverse effects would last for
at least 35 half-lives of the drug, which is anywhere
from 5 to 8 months. Although therapeutic levels may
fall off substantially by then, 1 month is too soon to
expect the effects to subside. Even though some iodine
and amiodarone molecules will likely remain absorbed
in fat stores for years, if not for life, therapeutic levels
should not exist for longer than what is predicted by the
half-life.
10. Answer: D
This patient is experiencing QT prolongation with sotalol, placing the patient at an increased risk of developing life-threatening torsades de pointes. Sotalol should
be immediately discontinued. Given the QT prolongation that occurred with sotalol, the same will likely occur
with dofetilide. Amiodarone is associated with minimal
risk of QT prolongation and thus would be an appropriate alternative agent to prevent ventricular arrhythmias.

ACCP Updates in Therapeutics 2013: The Pharmacotherapy Preparatory Review and Recertification Course
2-247

Cardiology I

11. Answer: A
Hypertensive emergency should be treated immediately by a 25% reduction in MAP, followed by a slow
reduction to goal for 57 days. The patients comorbidities guide the optimal therapy. His dialysis and SCr of
11.4 mg/dL are a contraindication to sodium nitroprusside caused by possible thiocyanate toxicity. Labetalol
(-blockers in general) is controversial in patients who
have taken cocaine, but its nonselective nature makes it
an option; however, a reduction of 50% initially is too
rapid a decrease in BP for safety. Clonidine is not an appropriate drug for hypertensive emergency because its
unpredictable oral nature is difficult to titrate and can
lead to precipitous drops in BP beyond the goal 25% reduction and possibly stroke or worsening MI. Nitroglycerin is an optimal choice, considering the patients lack
of contraindications to this therapy and his evolving MI.

14. Answer: A
Epoprostenol is now warranted to manage this patients
underlying disease because he has not responded to two
oral PAH therapies and is now considered high risk because of the presence of right ventricular dysfunction
and low CI. The patient has normal filling pressures for
a patient with right ventricular dysfunction, and diuresis with furosemide may only worsen his low CI. The
underlying cause of his low CI is not elevated arterial
resistance; thus, nitroprusside would likely worsen his
hypotension. Correcting the elevated pulmonary pressures should correct the low CI; thus, dobutamine is not
indicated at this time, and it would likely only worsen
his tachycardia.

12. Answer: C
In the setting of asymptomatic hypertensive crisis (without acute target-organ damage), intravenous medications, as in Answer A and Answer B, plus admission to
the hospital are unnecessary. This patient is likely presenting because of a history of nonadherence. Resuming
her home medications (Answer C) at this time would be
most appropriate, with close follow-up to ensure that her
prescribed regimen is working. Adding a fourth agent
(Answer D) at this time is unnecessary, considering that
she could be controlled on her current drug regimen if
she were adherent. Follow-up should occur within the
first few days, rather than wait 7 days.
13. Answer: C
This patient is already receiving therapy with calcium
channel blockers to control her HR caused by AF. She
is taking a considerable dose of diltiazem, and her HR
is unlikely to tolerate further increases in therapy. Sildenafil is indicated for functional class I patients to improve symptoms or for patients whose other therapies
have failed. Although bosentan is an attractive oral option to manage her PAH, her liver enzymes are elevated
more than 3 times the upper limit of normal. In this setting, administering bosentan is not recommended. If
liver transaminases are elevated transiently because of
hepatic congestion, bosentan may be reconsidered later.
Because this patient is currently in functional class IV
with symptoms at rest, epoprostenol is indicated for a
survival benefit.

ACCP Updates in Therapeutics 2013: The Pharmacotherapy Preparatory Review and Recertification Course
2-248

Cardiology I

ANSWERS AND EXPLANATIONS TO SELF-ASSESSMENT QUESTIONS


1. Answer: C
In this patient, the characteristic of chest pain, presence of
ST-segment depression on ECG, and positive biomarkers
for myocardial necrosis suggest NSTEMI. Because of his
presentation characteristics, he is at a high-enough risk
to warrant cardiac catheterization (invasive strategy).
This invasive strategy is used to determine whether occluded or partly occluded epicardial arteries exist, which
ones can be intervened on, and whether to make a PCI
(percutaneous transluminal coronary angioplasty with or
without stenting). Initial therapy for ACS usually consists
of morphine, oxygen, nitroglycerin, and ASA (MONA)
therapy, but only ASA has been shown to reduce mortality from these initial treatments. Aspirin should be given
as soon as possible after hospital presentation and continued indefinitely, if tolerated. On the basis of clinical trials, guidelines, and experience, an initial dose of 162325
mg is recommended (Answer B and Answer D are not
the best choices regarding dosing for an acute episode).
Aspirin coupled with a P2Y12 receptor antagonist is indicated for an early invasive strategy in the management
of UA/NSTEMI, reducing outcomes. The 2012 UA/
NSTEMI focused update gives a class I recommendation for clopidogrel, ticagrelor, and prasugrel in the ACS
setting in patients undergoing PCI. The choice of which
P2Y12 receptor antagonist in the ACS setting depends on
patient presentation, contraindications, and whether PCI
is involved; in this case, any of the three P2Y12 antagonists would be appropriate. The anticoagulation strategy
treatment for ACS generally includes one anticoagulant
(UFH, LMWH, fondaparinux, or bivalirudin). When
UFH is chosen as an anticoagulant strategy, the dose
used for ACS is a 60-unit/kg bolus and 12-unit/kg/hour
infusion (Answer A is an incorrect dosing strategy). On
the basis of dosing, bivalirudin (Answer D) would be an
appropriate anticoagulation strategy; however, the initial
dose of ASA should be higher. As long as the patient has
no signs of HF, evidence of low output state, increased
risk of cardiogenic shock, or other contraindications to
-blockade, it is reasonable to give intravenous -blocker
therapy (Answer A); however, starting oral therapy within 24 hours is preferred. Intravenous ACE inhibition (Answer B) should not be given to patients within the first 24
hours of ACS because of the increased risk of hypotension.

2. Answer: B
Given that this patient experienced a significantly low
platelet count with his last recent heparin exposure and
suspected HIT, the use of any GPI would be unwise for
ACS treatment because these agents need to be combined with UFH. Thrombolytic therapy is not recommended for the treatment of UA/NSTEMI and would
not be appropriate in this patient. Bivalirudin, a direct
thrombin inhibitor, would be the treatment of choice in
patients with HIT undergoing PCI.
3. Answer: C
This patient is well perfused and can be classified as
Forrester hemodynamic subset II (warm and wet). Because the patient is congested (shortness of breath, dyspnea at rest), intravenous diuretics are first-line therapy.
Nitroglycerin is the best choice in this setting because
vasodilatory agents can be used in conjunction with intravenous diuretics to improve acute pulmonary edema
or severe HTN. When adjunctive therapy is required
in addition to loop diuretics, intravenous vasodilators
should be considered over inotropic agents. Dobutamine and milrinone primarily increase CO, which is
not a considerable problem in warm and wet exacerbations. In addition, the adverse effects of these agents (increased mortality, proarrhythmia) limit their use. Intravenous metoprolol should be used extremely cautiously
because of its negative inotropic effects and because this
patient is not in a euvolemic state.
4. Answer: D
This patient has a depressed LVEF less than 40%, so her
drug therapy options are limited. Procainamide is indicated only in secondary prevention of sustained VT in
patients with a normal LVEF greater than 40%; if given
to this patient, it could worsen her HF. Metoprolol is
indicated only for treating patients with asymptomatic
nonsustained VT and SVT associated with CAD. This
patient had an episode of sustained VT. Her QTc interval is not prolonged at 380 milliseconds, and her serum
magnesium level is within normal limits, so she does
not require intravenous Mg therapy. Amiodarone is firstline treatment of patients without contraindications because of its efficacy and safety in patients with an LVEF
less than 40%.

ACCP Updates in Therapeutics 2013: The Pharmacotherapy Preparatory Review and Recertification Course
2-249

Cardiology I

5. Answer: B
This patient shows target-organ damage from poorly
controlled HTN in the form of a cerebrovascular accident. Although fenoldopam is indicated for treating
hypertensive emergency, its use is cautioned in patients
with stroke symptoms because its dopamine agonist
activity can cause cerebral vasodilation and potentially
reduced blood flow to the ischemic areas of the brain.
Nicardipine is an appropriate choice for this patient because its calcium channel blocking effects will reduce
BP and potentially decrease vasospasm in the cerebral
arteries, which may lead to further ischemia or seizure
activity. Although labetalol is an effective option for
treating this patients hypertensive emergency, she has a
history of asthma and a low HR, making labetalol a lessthan-ideal option for treating her symptoms. The antihypertensive effects of enalaprilat depend on a patients
renin activity, which is unknown in this case. Therefore,
the BP-reducing effects may be more difficult to control than with a drug having a more consistent effect in
individuals. In addition, the bolus nature of the drug is
not ideal for tightly controlling BP with a 25% reduction in MAP. Continuous-infusion drugs are preferable
for easier titration to effect in a hypertensive emergency.
6. Answer: C
This patient is experiencing hypertensive urgency, considering that he has no evidence of target-organ damage.
Thus, his BP may be reduced over 24 hours using oral
medications. Given this patients concomitant comorbidities, HF, and microalbuminuria, an ACE inhibitor
would be indicated. Sublingual nifedipine is no longer
recommended for management of hypertensive urgency
because of acute BP lowering and association with lifethreatening adverse events such as MI and stroke. Clonidine and labetalol are acceptable options; however, the
patient has compelling indications for an ACE inhibitor.
Although the patient should receive a -blocker in addition to an ACE inhibitor for HF management, labetalol is not one of the three -blockers recommended for
chronic HF management.
7. Answer: C
The Cardiac Arrest Study Hamburg trial compared ICD
with antiarrhythmic therapy in survivors of cardiac arrest for secondary prevention of SCD. The propafenone
study arm was discontinued early because of a significantly (61%) higher mortality rate compared with ICDs.

Although this trial had a small sample size that prevented a statistically significant difference in total mortality from being shown in ICD-treated patients versus
patients treated with either amiodarone or metoprolol,
the incidence of sudden death was significantly reduced
in patients with an ICD (33% vs. 13%, p=0.005). The
AVID (Antiarrhythmics Versus Implantable Defibrillators) trial also evaluated ICD implantation versus antiarrhythmic drug therapy (primarily amiodarone) in
survivors of SCD. Patients with ICDs had a significantly higher rate of survival than those treated with drug
therapy (89% vs. 82%, p<0.02).
8. Answer: C
The number needed to treat can be calculated by 1/absolute risk reduction. Because the absolute risk reduction in
mortality at 60 months was 7.2% with ICD versus placebo, 1/0.072 would be used to calculate the number of
patients needed to treat to prevent one death during this
time. About 14 patients would need to be treated with
ICD to prevent one death in 60 months versus placebo.
Other calculations in this fashion, including relative risk
reduction and 100% minus the absolute or relative risk
reduction, do not provide useful information for interpreting the trial results and yield an incorrect number of
patients.
9. Answer: D
International Pharmaceutical Abstracts is a database of
primarily pharmaceutical abstracts in more than 750
journals, including foreign and state pharmacy journals,
in addition to key U.S. medical and pharmacy journals.
Many of the citations are not included on MEDLINE,
so a broader search can be performed; however, subject descriptors are not consistently defined in a uniform
way, and multiword terms are often cited backward. The
Iowa Drug Information Service database offers fulltext articles from 1966 to present in about 200 medical
and pharmacy journals (primarily based in the United
States). It is updated monthly, so it may take longer to
access newly available articles from this service. The
Clin-Alert database contains more than 100 medical and
pharmacy journals focused on adverse events, drug interactions, and medical-legal issues. It is used primarily
to look up adverse events (especially recent reports) associated with medications. Excerpta Medica is a comprehensive database of more than 7000 journals from
74 countries dating from 1974 to present. Recently pub-

ACCP Updates in Therapeutics 2013: The Pharmacotherapy Preparatory Review and Recertification Course
2-250

Cardiology I

lished articles appear in the system within 10 days of


article publication, and it often contains data not found
in a typical MEDLINE search.
10. Answer: B
MedWatch is a post-FDA approval program established
by the FDA for health care professionals to report the
adverse events that occur after a drug is approved.
Although it is commonly used only for reporting serious reactions to the FDA, it can be used to report any
adverse event. Information recorded on these forms
is reported to the manufacturer and used to determine
whether black box warnings are necessary or whether
new adverse effects are seen with a drug. The Joint
Commission requires that all institutions have a definition of an ADR for the institution that can be understood
and remembered by all health care professionals. In addition, the Joint Commission requires that each drug
dose administered be monitored for adverse effects,
that each institution have a system in place for reporting
ADRs, and that the institution ensure that the reporting
mechanism identifies all key ADRs.
11. Answer: B
Because the Pharmacy and Therapeutics Committee
wants to discover whether the new drug is worth the
extra cost for the added mortality benefits it can provide for patients with decompensated HF compared
with available therapies, a cost-effectiveness analysis is
the best pharmacoeconomic analysis to perform. Costminimization analysis is used to determine whether a
therapeutically equivalent drug within a class that provides a therapeutic outcome the same as others available
can be used for less cost. Cost-utility analysis is used to
determine whether a drug can improve the quality of a
patients life more than other available therapies. Costbenefit analysis is used to evaluate new programs or services to determine whether they provide enough benefit
to justify the cost of running the program.

ACCP Updates in Therapeutics 2013: The Pharmacotherapy Preparatory Review and Recertification Course
2-251

Cardiology I

ACCP Updates in Therapeutics 2013: The Pharmacotherapy Preparatory Review and Recertification Course
2-252

S-ar putea să vă placă și